«Бог играет в кости со Вселенной, не брезгая, впрочем, обманом. И сейчас главная цель физики состоит в том, чтобы выяснить, какими правилами руководствуется Всевышний, а затем использовать их в собственных целях» (Джозеф Форд)

«Мир, полный склонностей и случайных событий, но подчиняющийся хорошо определённым законам, кажется мне гораздо интереснее, чем мир, где всё было предопределено в точности с самого начала времён» (Николя Жизан)

«Самой большой ошибкой было бы решить, что раз мы биологические машины, мы ничего не можем изменить, что Вселенная на молекулярном уровне еще 5 миллиардов лет назад предопределила, что в какой-то момент вы плохо обойдётесь с каким-то человеком, причините ему вред или что-то в этом роде. Это уже не научный биологический детерминизм, а какая-то детерминистская религия — вера в предопределённость» (Ричард Докинз)

Миром правит детерминизм или случайность? Будущее предопределено или многовариантно? Можно ли в точности предсказать будущее состояние системы по начальным условиям, или её динамика в принципе непредсказуема? Всё происходит согласно необходимости и провидению, или наша судьба создаётся на ходу? Может ли Бог быть генератором случайных чисел? Или он всё-таки не играет в кости? Только не говорите, что вы никогда не задумывались над этими вопросами и не хотели бы узнать, как оно есть на самом деле. Даже проблема свободы воли на их фоне вторична, поскольку целиком зависит от проблемы причинности и детерминизма.

Разумеется, ответы следует искать в квантовой механике, ведь в основе всех физических явлений лежат законы движения и взаимодействия элементарных частиц. На сегодняшний день это самая фундаментальная и самая экспериментально подтверждённая теория из всех, которые у нас есть. У неё сложный математический аппарат, но он работает безотказно и даёт предсказания, сбывающиеся с точностью до тысячных знаков после запятой. Тем не менее, именно квантовая механика даёт понять, что будущее принципиально непредсказуемо, и никакая сила не может заранее предопределить ход событий. Как же так? Нет ли здесь противоречия? Может, учёные сами не знают, детерминированы физические процессы или случайны? Нет, на этот счёт наука даёт чёткий и недвусмысленный ответ. Всё, что вы прочитаете дальше – только конкретика, никакой философской неоднозначности.

Высший замысел, судьба, фатум и другие предопределённости

Среди нас есть люди, которые верят в судьбу. Они всерьёз думают, что всё в этом мире предопределено, случайности не случайны, и в каждом событии есть какой-то смысл. При этом, если сильно постараться, можно в точности предсказать будущее – например, дату и причину своей смерти. Им не понятно, что всеобщая предопределённость и возможность узнать будущее логически несовместимы, ведь само по себе знание будущего уже меняет судьбу, как самосбывающееся пророчество. Но каким-то образом вера в Провидение или Высший замысел сочетается у них с верой в астрологию, вещие сны, приметы, таро, хиромантию и прочие виды гадания. «Умеренных» детерминистов, которые допускают возможность изменить свою судьбу или хотя бы прийти к предопределённому событию разными путями, понять ещё можно. В конце концов, большинство предсказаний столь неоднозначны и расплывчаты, что под них можно подогнать практически любой возможный исход, чем обычно и пользуются грамотные астрологи и гадалки. Правда, в наши дни они проигрывают конкуренцию большим языковым моделям – в деле генерации словесной «воды» им нет равных.

Но как быть с фаталистами, искренне считающими, что всё на свете абсолютно детерминировано ещё с начала времён, и мы не в силах ни на шаг отступить от этого сценария? Конечно, с практической точки зрения их убеждения совершенно бессмысленны, поскольку ни один здравомыслящий человек не скажет, что знает всю свою судьбу до мельчайших деталей, а если её невозможно узнать, то зачем вообще говорить о судьбе? Лучше просто жить, как будто никакой судьбы нет, и не заниматься пустым философствованием. Ещё Аристотель выдвинул против фатализма т.н. «праздный аргумент»: если всё предопределено, то было бы бессмысленно или бесполезно предпринимать какие-либо усилия, чтобы это осуществить. Если вам суждено выздороветь от некой болезни, то вы выздоровеете независимо от того, позовёте ли вы врача или нет. Если вам суждено не выздороветь, вы не выздоровеете, вызовете ли вы врача или нет. Но вам либо суждено выздороветь от болезни, либо суждено не выздороветь, поэтому обращаться к врачу бесполезно.

Однако главная опасность фатализма даже не в том, что он вынуждает смириться со своей участью и плыть по течению. Зачастую как раз наоборот, уверенность в том, что вам суждено прожить столько-то лет и умереть по заранее известной причине приводит к рискованным поступкам. Вспомните фильм «Новейший Завет» (2015), где каждый человек на Земле получает на телефон СМС с точной датой смерти. В ответ один видеоблогер начинает испытывать судьбу, прыгая со всё большей высоты и каждый раз чудом оставаясь в живых. Похожий эпизод есть в третьем сезоне сериала «Мир Дикого Запада»: Долорес запускает революцию в мире людей, рассылая всем сообщения с прогнозом их будущего, вычисленным единым суперкомпьютером. «Все они едут на поезде. Мы покажем им рельсы». Можно ли в действительности сделать такой прогноз, мы скоро узнаем. А пока я хочу привести пример из нынешних реалий, чтобы показать, что фатализм убивает.

За два года жизни в прифронтовом городе мне приходилось слышать и видеть результаты применения почти всех современных видов неядерного оружия: удары баллистическими и крылатыми ракетами, артобстрелы, залповый огонь, кассетные снаряды, авиабомбы, дроны-камикадзе и т.д. У каждого из них своя степень точности и радиус возможного отклонения от цели, от которого зависит распределение вероятностей попадания снаряда в то или иное место. В городе есть более, а есть менее безопасные места, но в конечном итоге от «прилётов» не застрахован никто, и ваша жизнь зависит от незначительных колебаний сопротивления воздуха на пути следования ракеты. Многие не хотят играть в русскую рулетку и уезжают, но какая-то часть населения отказывается от эвакуации и остаётся дома даже там, где идут городские бои. Кто-то не желает бросать своё имущество и расставаться с близкими, кто-то ждёт «освобождения», а кто-то уверен: если ему суждено погибнуть от обстрела, то судьба настигнет его где угодно, а если нет – он выживет даже при прямом попадании. Вроде бы никто и не готов добровольно принять смерть от вражеского огня, расстаться с жизнью по воле случая или по собственной глупости, другое дело – когда так предначертано высшими силами. В качестве примера они приводят многочисленные случаи гибели переселенцев даже вдали от линии фронта и чудесного спасения из-под завалов целыми и невредимыми людей, оказавшихся в эпицентре взрыва.

Я не буду здесь давать психологические нравоучения о нежелании брать ответственность за свою жизнь. Также не буду лишний раз напоминать об ошибке выжившего и других когнитивных искажениях, из-за которых мы недооцениваем вероятность случайных совпадений и не видим статистических закономерностей. И о свободе воли мы пока рассуждать не будем. Моя цель в другом – разобраться, насколько оправдана с точки зрения физики безусловная вера в судьбу или «высший замысел», отключающая инстинкт самосохранения.

Детерминизм в религии и философии

Возможно, идея предопределённости будущего заложена в нас эволюцией. Все сложные расчёты мозг проводит бессознательно и принимает решение без участия разума, отчего последний склонен верить, что им управляют духи или боги. Мифологические системы, религиозные доктрины и философские учения древности в большинстве своём предполагали существование судьбы, которой подчинена история мира в целом и жизнь отдельного человека в частности. Как правило, судьба считалась единоличным божественным провидением или выражением воли многих богов, хотя некоторые адепты (например, язычники Древней Греции и Рима) верили, что даже боги не могут избежать предписанной им судьбы. Тем не менее, с абсолютным фатализмом соглашались далеко не все. Вере в судьбу обязательно сопутствовала вера в возможность её узнать путём всевозможных гаданий, изучения движения небесных тел и т.д. А если предсказания не сулили ничего хорошего, допускалась возможность изменить судьбу собственными действиями. Что касается судьбы Вселенной, то она всецело отдавалась в руки высших сил.

Пифия - дельфийский оракул
Пифия - дельфийский оракул

В восточной философии фатализм практически отсутствует. Понятие кармы гораздо ближе к причинному детерминизму, поскольку «карма» переводится как «действие» и включает причинно-следственные связи между поступками и результатами, сохраняющиеся в процессе реинкарнации. Плохие поступки в прошлой жизни создают плохую карму в жизни настоящей и не дают выйти из колеса Сансары, а хорошие поступки создают хорошую карму, направляющую к просветлению. Самой первой и наиболее проработанной теорией причинности стала буддийская теория взаимозависимого возникновения – пратитья-самутпада. Одна из её формулировок звучит так: «если есть то, значит, есть и это»; «если нет этого, значит, нет и того»; «когда есть это, то есть и то, если возникает это, то возникает и то, если это исчезает, то исчезает и то». Очевидно, буддисты пришли к пониманию, что у одной причины может быть множество следствий и у одного следствия – множество причин, и что необходимо объединённое взаимодействие различных причинных факторов или условий для формирования нужного результата.

Вера в судьбу как абсолютную предопределённость и неизбежность происходящего называется фатализмом. Она предполагает, что человек не может повлиять на будущее или на результат собственных действий, поэтому следует смириться с неотвратимым, а не сопротивляться ему. Различают теологический фатализм (свободная воля несовместима с существованием всеведущего Бога, который знает всё будущее), логический фатализм (утверждения о будущем, которые мы считаем в настоящее время истинными или ложными, могут быть истинными или ложными только в том случае, если будущие события уже предопределены) и причинный детерминизм (каждое последующее состояние системы определяется предыдущим, а не окончательным состоянием системы). В основе последнего лежит принцип причинности или каузальности, предполагающий, что каждое событие является следствием некой причины в прошлом и в свою очередь причиной некого следствия в будущем. Причина (лат. causa) — то, без чего не было бы следствия.

Детерминизм предполагает, что есть только одно, точно заданное возможное будущее. Детерминация означает, что при воздействии одного объекта (причина) происходит необходимое ожидаемое изменение другого объекта (следствие). Любое событие может оказывать влияние только на события, происходящие позже него и не может оказывать влияние на события, произошедшие раньше него. В отличие от фатализма, детерминизм предполагает наличие причинно-следственных связей, которые можно обнаружить и предсказать по ним будущее. Для фаталиста всё изначально предопределено судьбой или Высшим замыслом, а детерминист верит, что каждое событие имеет конкретную причину в прошлом, по которой всё происходит именно так, а не иначе. Фатализм принимает будущие события как неизбежные и полностью отрицает возможность повлиять на них своими действиями. Детерминизм не отрицает, что действия человека влияют на будущее, но сами действия человека определяются причинно-следственной цепочкой предшествующих событий, поэтому свободы воли как таковой нет.

Детерминизм в классической механике

Наука поначалу соглашалась с христианским учением о провидении и теологической идеей перводвигателя: достаточно было однажды привести Вселенную в движение, чтобы определить всю её судьбу наперёд. Так, в основе ньютоновской механики лежал принцип абсолютного детерминизма. Его суть отражала идея демона Лапласа – воображаемой сущности с неограниченной памятью, которая может знать положение и импульс каждой частицы во Вселенной в каждый отдельный момент времени:

«Мы можем рассматривать настоящее состояние Вселенной как следствие его прошлого и причину его будущего. Разум, которому в каждый определённый момент времени были бы известны все силы, приводящие природу в движение, и положение всех тел, из которых она состоит, будь он также достаточно обширен, чтобы подвергнуть эти данные анализу, смог бы объять единым законом движение величайших тел Вселенной и мельчайшего атома; для такого разума ничего не было бы неясного и будущее существовало бы в его глазах точно так же, как прошлое» (Пьер-Симон Лаплас)

Демон Лапласа
Демон Лапласа

То есть демон Лапласа мог бы по настоящему моменту узнать всю историю Вселенной и предсказать всё её будущее. «Дайте мне начальные данные частиц всего мира, и я предскажу вам будущее мира» - перефразировал Лаплас Архимеда. В философии Лапласа случайность — это «всего лишь выражение человеческого невежества». Вселенная работает как часовой механизм, каждое событие запускает следующее, как в каскаде падающих костяшек домино. Как писал Поль-Анри Гольбах,

«Ничего в природе не может произойти случайно; всё следует определённым законам; эти законы являются лишь необходимой связью определённых следствий с их причинами... Говорить о случайном сцеплении атомов либо приписывать некоторые следствия случайности — значит говорить о неведении законов, по которым тела действуют, встречаются, соединяются либо разъединяются».

Итак, с точки зрения физики детерминизм – это когда состояние системы в будущем полностью определяется её состоянием в прошлом. Если вы хорошо знаете начальные условия (например, положение и скорость объекта, силу тяжести, сопротивление воздуха и др.) и законы физики (например, законы движения Ньютона), вы можете в точности предсказать, что произойдёт с системой в любой последующий момент времени. Классическая механика допускала возможность такого предсказания. Даже если система слишком сложная, такая, как человек, и мы не можем на практике узнать все его переменные в конкретный момент времени, всё равно его будущее полностью детерминировано. Теоретически все его действия и даже мысли предсказуемы, просто нам понадобится демон Лапласа, обладающий неограниченной памятью, чтобы запомнить всю информацию о человеке на текущий момент, или его современный эквивалент – суперкомпьютер с неограниченной вычислительной мощностью. Из ньютоновской механики следовало, что нет никакой разницы между протеканием процесса вперёд во времени или назад – любое движение обратимо.

Причинный детерминизм
Причинный детерминизм

Термодинамика и статистическая механика добавили в ньютоновскую картину мира туман неопределённости. Движение каждой отдельной частицы полностью детерминировано, но, когда частиц много, их поведение становится хаотичным и непредсказуемым. В качестве меры беспорядка ввели специальную величину – энтропию. Она пропорциональна числу микросостояний, неразличимых с макроскопической точки зрения, то есть фактически отображает степень нашего неведения о системе. Согласно второму началу термодинамики, в изолированной системе энтропия не может уменьшаться, поэтому время для такой системы необратимо. Хотя необратимость эта чисто статистическая, и в принципе стрелу времени можно развернуть, если поменять импульсы всех частиц на противоположные. Почему это нельзя сделать на практике, я рассказывал в предыдущей статье «Стрела времени».

Проблема трёх тел, моделирование движения двойного маятника, прогноз погоды, предсказание извержений вулканов и землетрясений – все эти задачи считаются трудновычислимыми из-за чувствительности соответствующих систем к начальным условиям. Но законы термодинамики и статистической механики не исключают демона Лапласа, просто требуют от него огромной вычислительной мощности: чтобы предсказать динамику броуновского движения в стакане воды, ему нужно узнать положения и скорости порядка 1023 молекул. В таком случае он будет знать все детали системы с бесконечной точностью, вариаций в начальных условиях не будет, и статистическая механика вместе с теорией хаоса остаются неприменимыми – для описания движения отдельных частиц достаточно простых законов Ньютона.

Случайность в квантовой механике

Квантовая механика перевернула всё с ног на голову. Оказалось, что нельзя просто так взять и узнать одновременно положение и импульс частицы. Это невозможно в принципе, поскольку координата и импульс являются некоммутирующими операторами. Из-за принципа квантовой неопределённости Гейзенберга при каждом измерении сохраняется не полная информация о состоянии частицы, а только вероятности её местоположения и импульса. Стреляя одиночными электронами через две щели, мы не можем предсказать, в какую точку на экране попадёт частица. Но если сделать это много раз, на экране начнёт проявляться интерференционная картина. Электрон вероятнее попадёт в то место, где волны приходят в фазе, чем туда, где они приходят в противофазе. То есть можно определить вероятность обнаружения частицы в том или ином месте, но где именно она окажется – дело случая.

Демон Лапласа против неопределённости Гейзенберга
Демон Лапласа против неопределённости Гейзенберга

В квантовой механике состояние системы описывается волновой функцией, которая позволяет узнать распределение вероятностей получить некоторый результат при измерении, возведя модуль волновой функции в квадрат по правилу Борна. Но в самом акте измерения есть доля нередуцируемой, фундаментальной случайности. Например, мы измеряем спин электрона, который находится в суперпозиции состояний «вверх» и «вниз». Волновая функция предсказывает, что с вероятностью 30% будет получен первый результат и с вероятностью 70% - второй. Это не субъективные (байесовские) вероятности, выражающие степень нашего незнания о системе, а объективные характеристики этой системы. Если произвести такое измерение 100 раз, то примерно в 30-ти случаях спин будет направлен вверх, и в 70-ти случаях – вниз. Но при каждом конкретном измерении результат предсказать невозможно, это нередуцируемая случайность. Следовательно, но о каком детерминизме не может быть и речи, и будущее фундаментально непредсказуемо.

Истинная случайность

Но это всё квантовая теория, не имеющая никакого отношения к реальной жизни. Поэтому вы вправе спросить: какие у нас есть вещественные доказательства тому, что детерминизм невозможен? И получите ответ: вы можете прямо сейчас взять и купить это доказательство в интернет-магазине. Его название – квантовый генератор случайных чисел. Выглядит вот так:

На фото – первый коммерческий квантовый генератор случайных чисел производства швейцарской компании ID Quantique. Внутри у него есть источник света, излучающий единичные фотоны, которые проходят через полупрозрачное зеркало и попадают на один из двух детекторов. Каждый детектор ассоциирован с двоичным числом (битом). Фотон «выбирает» детектор истинно случайным образом, и в итоге у вас получается истинно случайное число.

Здесь необходимо прояснить понятие истинной случайности. Как известно, «генерация случайных чисел слишком важна, чтобы оставлять её на волю случая» (Роберт Кавью), но, с другой стороны – «всякий, кто питает слабость к арифметическим методам получения случайных чисел, грешен вне всяких сомнений» (Джон фон Нейман). С точки зрения теории информации случайность – это отсутствие определённой закономерности или предсказуемости в строке битов, мера неопределённости результата, и она имеет максимальную энтропию по Шеннону. Случайная последовательность символов не упорядочена и не следует понятной схеме или комбинации. Отдельные случайные события по определению непредсказуемы, но если известно распределение вероятностей, то частота различных результатов в повторяющихся событиях предсказуема.

Бог играет в кости
Бог играет в кости

Ваш компьютер, в целом работающий по законам классической физики и являющийся детерминистичной системой, может генерировать т.н. псевдослучайные числа. Например, когда вам нужно создать хорошо защищённый пароль, он выдаст на первый взгляд случайную последовательность символов, созданную детерминированным алгоритмом. Для такой цели можно ограничиться и этим, но нужно учитывать, что сгенерированная последовательность не является истинно случайной. Отношение между одним псевдослучайным числом и следующим предопределено, хотя предугадать его достаточно трудно. Разница ощущается тогда, когда необходимо осуществить численное моделирование какого-нибудь сложного объекта или процесса. Например, при разработке самолёта инженеры моделируют его на компьютере, чтобы не строить десятки прототипов и не испытывать их в аэродинамической трубе. Для симуляции условий полёта, где воздушные потоки могут непредсказуемо меняться, используют псевдослучайные числа. Но потом выясняется, что прототип самолета, который прекрасно вёл себя при таком моделировании, в реальности летает довольно плохо. А если использовать в симуляции истинно случайные числа, проблем не возникает.

Скрытые параметры

Конечно, упрямый и педантичный критик, солидарный с Эйнштейном в его неприятии квантовой механики, может и засомневаться в наших доводах. А что, если квантовый генератор случайных чисел на самом деле тоже генерирует псевдослучайные числа, просто наши измерительные приборы несовершенны и не могут предсказать результат? Возможно, за квантовой неопределённостью скрывается детерминированная классическая реальность, а нам просто не хватает знаний и точности приборов, чтобы рассчитать траектории частиц. Это так называемая теория скрытых параметров, предполагающая, что «выбор» фотоном детектора каким-то образом заранее предопределён. Мы уже кратко изложили историю опровержения теории скрытых параметров в статье «Жуткое дальнодействие». Всё началось в 1926 г. с переписки, в которой Альберт Эйнштейн заявил, что «Бог не играет в кости», а Нильс Бор ответил: «Не говорите Богу, что ему делать». В 1935 г. вышла статья Эйнштейна, Подольского и Розена с критикой квантовой механики и формулировкой т.н. ЭПР-парадокса. В 1964 г. Джон Белл формализовал проблему в виде знаменитых неравенств, которые выполнялись при наличии скрытых параметров и нарушались в случае их отсутствия. Ну и далее последовали эксперименты над квантово запутанными частицами, за которые Алан Аспе, Джон Клаузер и Антон Цайлингер получили Нобелевскую премию по физике 2022 г.

Напомню, что большинство экспериментов по устранению лазеек Белла проводились по единой схеме: источник излучает в противоположных направлениях два запутанных фотона, которые находятся в суперпозиции четырёх возможных состояний. Алиса и Боб независимо друг от друга и не «сговариваясь» предварительно с фотонами или их источником каждый в своей лаборатории выбирают, под каким углом измерять поляризацию. Фотоны проходят каждый через свой поляризатор и попадают на детектор, сообщающий экспериментатору результат измерения. Когда Алиса и Боб сверяют данные, результаты всегда получаются коррелированными: если выбор угла поляризации совпал – они строго противоположны, если не совпал – случайны. Следовательно, до момента независимого измерения состояния каждой частицы их можно описать только вероятностями, никаких скрытых параметров они не содержат. Вскоре после реализации ключевого эксперимента Аспе 1982 г., в 1986 г. сэр Джеймс Лайтхилл написал следующее:

«… я снова должен остановиться и сделать заявление от имени обширного глобального сообщества людей, работающих в области механики. Сегодня мы все глубоко уверены, что энтузиазм наших предшественников по отношению к изумительным достижениям механики Ньютона привел их к обобщениям в области предсказуемости, в которые мы были действительно склонны верить вплоть до 1960-х гг., но которые, как мы сейчас поняли, являются ложными. Мы хотим принести коллективные извинения за то, что вводили в заблуждение широкие слои образованного населения, распространяя идеи детерминизма систем, подчиняющихся законам движения Ньютона, которые после 1960 были доказательно опровергнуты».

Так вот, на сегодняшний день учёные убедительно доказали, что никаких скрытых параметров не существует. Если не верите, вам придётся признать, что действия 100000 добровольцев со всего мира, принимавших участие в онлайн-игре в ходе Большого эксперимента Белла 2016 г., были полностью детерминированы ради того, чтобы учёные собрали полученные ими случайные числа и получили в ходе опыта якобы «случайный» результат измерений. Более того, вам нужно будет объяснить, каким образом два квазара могли «сговориться» не позднее 7.8 млрд лет назад послать учёным спектральные данные, которые достигли Земли в 2018 г. и были использованы в эксперименте «Космический тест Белла», в очередной раз показавшем отсутствие скрытых параметров. После этого в детерминизм могут верить только конспирологи, допускающие возможность сговора учёных с квазарами ради сокрытия от нас истинной природы реальности, или фаталисты, утверждающие, что судьба каждой элементарной частицы была предопределена в момент Большого взрыва, ещё и предопределена таким образом, чтобы мы никогда не смогли этого доказать. Современная версия фатализма называется «супердетерминизм», и у неё даже есть сторонники среди известных учёных.

Супердетерминизм

Эксперименты по нарушению неравенств Белла вынуждают учёных отказаться от одного из двух фундаментальных принципов: реализма (веры в то, что объекты имеют определённые и присущие им свойства за пределами вашего наблюдения) и локальности (идеи о том, что вещи влияют друг на друга посредством прямого взаимодействия). Но для тех, кто не готов пожертвовать ни тем, ни другим, остаётся третий, ещё более удручающий вариант – супердетерминизм, постулирующий отсутствие статистической независимости между наблюдателем, когда его детектор взаимодействует с частицей при измерении, и самой частицей. Иначе говоря, Вселенная настолько детерминирована, что она вынуждает любое, казалось бы, случайное, свободно выполненное измерение давать значение, коррелирующее с другими измерениями.

Джон Белл
Джон Белл

Идею супердетерминизма впервые высказал в 1985 г. в радиоинтервью BBC Джон Белл:

«Есть способ избежать вывода о сверхсветовых скоростях и жутких действиях на расстоянии. Но он предполагает абсолютный детерминизм во Вселенной, полное отсутствие свободы воли. Предположим, что мир супердетерминирован, и не только неживая природа работает по закулисному часовому механизму, но и наше поведение, включая нашу веру в то, что мы свободны в выборе проведения одного эксперимента, а не другого, абсолютно предопределено, включая "решение" экспериментатора провести одну серию измерений, а не другую, и затруднение исчезает. Нет необходимости в сигнале, превышающем скорость света, чтобы сообщить частице A, какое измерение было проведено с частицей B, потому что Вселенная, включая частицу A, уже «знает», каким будет это измерение и его результат».

В 1988 г. Карл Бранс предложил первую супердетерминированную модель скрытых переменных. Более свежими работами являются публикации Майкла Холла, Тима Палмера и Сабины Хоссенфельдер. Последняя известна ещё и как видеоблогер, она популяризует на своём канале квантовую физику и называет свободу воли "логически бессвязным бредом". Другое направление супердетерминизма – модели Вселенной на основе клеточных автоматов, развиваемые Герардом Хофтом и Стивеном Вольфрамом. Они сравнивают Вселенную с игрой «Жизнь» Конвея, в которой комбинация всего четырёх простых правил создаёт закономерности, напоминающие реальные жизненные ситуации. Об этих теориях я расскажу как-нибудь в другой раз.

Супердетерминизм решает проблему измерения в рамках локального реализма, не прибегая к солипсизму квантового байесианства, параллельным мирам Эверетта или жуткому дальнодействию механики Бома. Но это делается ценой полного исключения свободы воли и, что ещё важнее – возможности когда-либо узнать, что же на самом деле предопределено. Каждое наше действие, в том числе выбор экспериментаторами базиса измерения, предопределено начальными условиями на момент Большого взрыва. Если классический детерминизм предполагает локальные причинно-следственные связи между определёнными событиями в прошлом и будущем, то квантовый супердетерминизм прячет концы в воду, протягивая по крайней мере некоторые из этих причинно-следственных цепочек за горизонт событий наблюдаемой Вселенной. Другие модели супердетерминизма допускают даже ретропричинность – влияние будущих событий на прошлые.

Квазары, запутанные частицы и экспериментаторы в "Космическом тесте Белла" действовали коррелировано согласно инструкциям, предопределённым Большим взрывом
Квазары, запутанные частицы и экспериментаторы в "Космическом тесте Белла" действовали коррелировано согласно инструкциям, предопределённым Большим взрывом

По словам физика Антона Цайлингера, если супердетерминизм верен, некоторые из его следствий поставят под сомнение ценность самой науки, разрушая принцип фальсифицируемости:

«Мы всегда неявно предполагаем свободу экспериментатора… Это фундаментальное допущение необходимо для занятий наукой. Если бы это было не так, то, я полагаю, вообще не имело бы смысла задавать вопросы природе в эксперименте, поскольку тогда природа могла бы определить, каковы наши вопросы, и это могло бы направлять наши вопросы таким образом, чтобы мы пришли к ложной картине природы»

Действительно, гипотеза супердетерминизма принципиально нефальсифицируема, поскольку она постулирует, что корреляции существуют со времён Большого взрыва. Любая возможность узнать будущее означала бы вмешательство в этот механизм и переписывание сценария, что по определению невозможно. Но главная проблема супердетерминизма в том, что ранняя Вселенная, радиус горизонта событий которой не превышал планковской длины, должна была каким-то образом хранить в себе информацию о всех последующих событиях. Такое количество информации просто не могло поместиться в столь ограниченном объёме пространства – ограничение накладывает предел Бекенштейна.

На мой взгляд, супердетерминизм – это объяснение даже менее разумное, чем классическое «на всё воля божья». Конечно, оно может понравиться конспирологам и фаталистам, но квантовый заговор гораздо хуже сговора масонов с рептилоидами или уготованной каждому из нас судьбы. От супердетерминированного заговора не просто нет спасения, вы даже никогда не сможете отличить его от случайности. Какие бы вы не ставили эксперименты по его раскрытию, их результаты всегда будут коррелировать с вашими действиями именно таким образом, чтобы выглядеть случайными. Кем бы ни был автор этой программы, он имеет мало общего со старым-добрым монотеистическим Богом, который не стал бы лишать человека свободы воли и намеренно его обманывать. Скорее он похож на злого демона Декарта, специально вводящего вас в заблуждения, транслируя вашему разуму виртуальную реальность. Хотя супердетерминисты и считают себя последователями Эйнштейна, сам Эйнштейн вряд ли принял бы эту теорию. Он бы сказал: «Бог изощрён, но не злонамерен». Поэтому Говард М. Уайзман и Эрик Кавальканти справедливо утверждают, что любая гипотетическая супердетерминистская теория «была бы примерно такой же правдоподобной и привлекательной, как вера в повсеместный инопланетный контроль над разумом».

Вероятностный (ин)детерминизм

Итак, мы выяснили, что существует фундаментальная, подтверждённая на практике разница между истинно случайными и псевдослучайными числами. Попытки Эйнштейна и других учёных доказать, что «Бог не играет в кости», провалились. Но всё же квантовомеханическая картина мира не исключает детерминизма, просто речь идёт о детерминированных вероятностях. Математическим основанием этого квантового детерминизма служат принципы CPT-симметрии и унитарности операторов, которые я разбирал в статье «Квантовая информация и законы сохранения».

В чём разница между квантовым измерением и броском игральной кости? В первом случае мы получаем истинно случайный результат, а во втором итог предопределён уже на стадии вращения кубика, просто вычислить его мы не можем. Однако истинная случайность не исключает, что у события есть причина в прошлом. Вот что пишет об этом Николя Жизан в книге «Квантовая случайность»:

«Истинная случайность, таким образом, не имеет причины в том смысле, который подразумевается в классической физике. Результат истинного случая никоим образом не предопределён. Но тут необходимо уточнить, что событие, выглядящее как истинно случайное, может иметь причину. Дело лишь в том, что эта причина определяет не результат, а только предрасположенность к реализации определённого результата»

А вот как Жизан объясняет разницу между классическим и квантовым детерминизмом:

«В классической физике результат любого измерения предопределён. В определённом смысле он записан в физическом состоянии системы, которую мы измеряем. Вероятности вступают в игру только из-за того, что точное физическое состояние нам неизвестно… В квантовой физике результат измерения не предопределён, даже если состояние системы нам хорошо известно. В физическое состоянии системы, которую мы измеряем, записана лишь её склонность к проявлению того или иного результата»

«Тем не менее, следует заметить, что некоторые результаты в квантовой физике предопределены. Структура математической теории квантовой физики (гильбертово пространство) такова, что так называемых чистых состояний, в которых нет места незнанию, набор всех предопределённых результатов однозначным образом характеризует предрасположенность ко всем остальным возможным результатам. В этом смысле предрасположенности квантовой физики являются логическим обобщением классического детерминизма»

Здесь речь идёт об уравнении Шрёдингера, описывающем эволюцию волновой функции. Суперпозиция всех возможных состояний частицы в один момент времени однозначно предопределяет такую же суперпозицию в другой момент. Случайность вступает в игру только при квантовом измерении, когда чистое состояние переходит в смешанное, то есть объект измерения запутывается с измерительным прибором. Как мы уже писали, при каждом квантовом измерении (грубо говоря, при любом взаимодействии элементарных частиц) реализуется одно из множества возможных состояний (соотношений координаты и импульса) измеряемой частицы. Вероятность реализации каждого из этих состояний зависит от квантовых свойств частицы, её предыдущих взаимодействий, степени запутанности с частицами, с которыми она ранее взаимодействовала, и т.д. Например, вероятность первого состояния частицы – 50%, второго – 33%, третьего – 16%, четвёртого – 1%. То, что первое состояние наиболее вероятно, не значит, что при данном измерении реализуется именно оно, но, если провести аналогичное измерение сто раз, примерно в половине случаев мы получим первое состояние.

Критики детерминизма ссылаются на принцип неопределённости Гейзенберга – мол, в квантовой механике нельзя одновременно узнать координату и импульс частицы, значит, её поведение абсолютно случайно. Это ошибочный вывод. На самом деле уравнение Шрёдингера, описывающее эволюцию волновой функции частицы, полностью детерминистично. Но этот детерминизм отличается от детерминизма ньютоновской механики. В классическом понимании событие полностью предопределено, когда его вероятность равна 100%. В квантовой механике такое тоже бывает, с той лишь разницей, что если мы на 100% будем знать координату частицы, мы не будем ничего знать о её импульсе, и наоборот. Но в большинстве случаев речь идёт о каком-то соотношении вероятностей, зная которое, можно спрогнозировать развитие системы как вперёд во времени, так и назад. Иначе говоря, сохраняется не конкретная классическая информация о частице, а вероятностная квантовая, или степень неопределённости состояния.

Квантовый детерминизм – очень трудная для понимания и контринтуитивная идея. В повседневной жизни мы привыкли к тому, что информация не сохраняется. Грубо говоря, если мы складываем 2+2 и получаем 4, по этой четвёрке уже нельзя сказать, каким путём она образовалась: 2+2, 3+1 и т.д., значит, операция необратима. Казалось бы, то же самое и с частицами. Вот у вас есть атом урана-235, который имеет период полураспада 700 млн. лет. Единственное, что вы можете предсказать – что за это время он с вероятностью 50% распадётся на торий-231 и гелий-4, но когда именно это произойдёт – неизвестно. Или вы сталкиваете протон с протоном суммарной энергией 2 ТэВ, и у вас с какой-то вероятностью может родиться пара топ-кварков, а может – пара W- и Z-бозонов, которые опять-же с какой-то вероятностью образуют бозон Хиггса. Всё, на что способны учёные – вычислить те самые вероятности, но каким будет исход конкретного столкновения – неизвестно. Разве это не случайность?

Квантовая случайность не исключает наличия причинно-следственных связей между событиями, просто эти связи становятся не детерминированными, а неустранимо вероятностными. Например, событие А с вероятностью 77% приводит к событию Б, а с вероятностью 23% - к событию В. Индетерминированная (или вероятностная) причинность - это предполагаемая возможность, когда что-то может иметь необходимую причину, а может и не иметь, а необходимое условие может приводить к одному неизбежному результату, а может и не приводить. Из утверждения «всё имеет причину» не следует утверждение «всё, что произойдёт, предсказуемо». Если никто не может просчитать события быстрее, чем они разворачиваются, то они поистине непредсказуемы. Вопрос «повторилось бы всё в точности так же, если бы Вселенная была переиграна заново с нуля?» не имеет смысла, поскольку вероятность такого повторения исчезающе мала, а при малейших отличиях в начальных условиях это была бы уже другая Вселенная. Каждое очередное квантовое измерение направляет процесс эволюции Вселенной по неизвестной заранее траектории.

Случайность и детерминизм в интерпретациях квантовой механики

В копенгагенской интерпретации процесс измерения сопровождается коллапсом волновой функции, суперпозиция разрушается, и мы получаем смешанное состояние измеренной частицы и запутанного с ней детектора. Когда именно и как происходит этот коллапс, интерпретация не объясняет. Ясно только, что результат измерения совершенно случаен, причём случайность эта фундаментальная и нередуцируемая. Отсюда следует, что информация об альтернативных состояниях теряется, по крайней мере для наблюдателя. Но индетерминизм, постулирующий истинную случайность квантовых событий и открытость будущего, противоречит унитарным уравнениям квантовой механики, согласно которым информация не может быть создана из ничего или уничтожена. Даже если считать волновую функцию и её коллапс субъективными предсказаниями агента, как это делают квантовые байесианцы, проблема потери информации остаётся. Для её решения физикам приходится прибегать к различных ухищрениям, вплоть до изменения всей математики квантовой теории.

Например, Николя Жизан, будучи сторонником индетерминизма, отвергает принцип сохранения информации и говорит, что «время реально идёт, а новая информация создаётся при измерении». В своих недавних статьях Жизан пытается обосновать эту позицию при помощи интуиционизма – направления в философии математики, отрицающего существование вещественных чисел с бесконечным количеством цифр после запятой. «Вещественные числа с бесконечным количеством знаков не могут иметь отношения к физике», поскольку информация – это физическая величина, и любой объём пространства имеет конечную информационную ёмкость согласно формуле Бекенштейна-Хокинга. Начальные условия Вселенной потребовали бы втиснуть слишком много информации в слишком маленький объём. Блок-вселенная, подразумевающая существование бесконечной информации, должна развалиться. Если же физические величины конечны и ограничены в точности, тогда природа становится по сути неточной и непредсказуемой. Но Жизан почему-то рассматривает информацию в рамках классической теории информации Шеннона, не принимая в расчёт квантовую теорию информации. Последняя допускает хранение бесконечного количества классической информации даже в одном кубите, однако позволяет извлечь при измерении этого кубита всего один классический бит.

Противоположный подход – многомировая интерпретация – требует признать, что нереализованные состояния из суперпозиции никуда не пропадают. Эти состояния тоже реализуются в результате измерения, только происходит это в параллельных вселенных. Например, в половине вселенных реализуется первое состояние, в трети вселенных – второе, в 16% - третье, и только в одной вселенной из ста – четвёртое. В Мультивселенной происходит всё, что физически может произойти. Вообще всё, в том числе взаимоисключающие события. Вероятность любого возможного события всегда равна 100%. Это возвращает к жизни демона Лапласа и даже упрощает ему работу. Квантовый демон Лапласа решает не уравнения движения Ньютона, а уравнение эволюции волновой функции Шрёдингера, параллельно вычисляя все возможные варианты развития событий. Для классического компьютера это непосильная задача, но, если демон Лапласа – квантовый компьютер, обладающий вычислительной мощностью наблюдаемой Вселенной, он смоделирует все варианты её прошлого и будущего. Правда, при измерении вы не сможете заставить его выдать именно тот результат, который реализуется на самом деле.

В теории Эверетта волновая функция не коллапсирует, а «растекается» в процессе декогеренции по всей вселенной. Но наш мозг этого не осознаёт, потому что мы сами находимся в суперпозиции всех своих возможных состояний, и каждый из наших двойников получает свой результат измерения. Формально многомировая интерпретация детерминистична, то есть в ней всё совершенно предсказуемо и происходит в соответствии с принципом сохранения информации. И то событие, которые в отдельной вселенной (синглвёрсе) кажется случайным, с позиции Мультивёрса становится полностью детерминированным. Но в какую именно вселенную (синглвёрс) попадёт «расщепившийся» наблюдатель в результате конкретного измерения, можно предсказать лишь с определённой долей вероятности. Поэтому многомировая интерпретация не имеет ничего общего с ньютоновским детерминизмом и на практике не даёт предсказаний, отличных от предсказаний традиционной квантовой теории. С точки зрения наблюдателя не имеет никакого значения, случаен сам результат измерения или то, в какой вселенной наблюдатель окажется по его итогу – в любом случае это случайность.

Как предсказать будущее?

Случайность случайностью – скажете вы, – но она же не мешает компьютерам, автомобилям и прочей технике работать вполне себе детерминированно, каждый раз отвечая на наши запросы предсказуемым образом? Может, это только физики своими экспериментами с котами Шрёдингера привносят в наш мир истинную случайность, а в остальном макроскопическая реальность полностью детерминирована, как предписывали Лаплас и Ньютон? Как бы ни так. Радиоактивный распад нестабильных изотопов, броуновское движение молекул жидкости, тепловые колебания атомов твёрдых тел – всё это источники квантового шума. Как правило, эти микроскопические флуктуации усредняются и не влияют на поведение макроскопических объектов. Но в хаотических системах вроде атмосферы, океана или плазмы в термоядерном реакторе возникает чувствительность к начальным условиям, более известная как эффект бабочки. Последние исследования показывают, что запускать этот механизм могут даже отдельные атомы и молекулы, распространяя квантовые флуктуации на макромасштабы. Неопределённость, случайность, хаос – неотъемлемые свойства нашего мира, не позволяющие заглянуть достаточно далеко в будущее и детерминировать движение элементарных частиц. Они проявляются везде, начиная с пресловутой проблемы трёх тел, не позволяющей точно рассчитать орбиты в гравитационном поле, и заканчивая аттрактором Лоренца, исключающим возможность предсказать погоду более, чем на 10 дней.

Будущее непредсказуемо
Будущее непредсказуемо

Разумеется, одни системы более подвержены влиянию квантовых флуктуаций, другие – менее. Карл Поппер в статье «Об облаках и часах» использовал часы в качестве примера детерминированной системы с регулярным и предсказуемым поведением, а облака – как пример хаотической недетерминированной системы. Все остальные объекты и процессы можно разместить где-то между этими полюсами: например, животных – ближе к облакам, растения – ближе к часам. В рамках лапласовского детерминизма даже облака являются часами, но с точки зрения современной физики скорее наоборот – даже самые точные часы иногда проявляют свойства облаков.

Так можно ли предсказать или спрогнозировать в этом мире хоть что-нибудь? Можно, если речь идёт о поведении несложной и более-менее изолированной системы. Например, в лаборатории можно легко приготовить фотон или электрон в определённом состоянии, в котором известны все параметры его волновой функции, и предсказать эволюцию его состояния во времени. Но у того же электрона нельзя знать проекцию спина на все 3 оси сразу – измерить её можно только по одной оси. Чтобы предсказать что-либо, нужно решить систему уравнений с заданными начальными условиями, а в точности узнать эти условия не всегда получается. В частности, теорема о запрете клонирования не позволяет узнать полное квантовое состояние системы, если она существует в единственном экземпляре. К тому же квантовую систему очень трудно смоделировать на обычном компьютере, поэтому понадобится квантовый компьютер. Современные квантовые процессоры имеют сотни кубитов, т.е. элементарных вычислительных ячеек. Такой процессор можно привести в некоторое состояние и затем измерить. Чтобы предсказать результат измерения, потребуется записать волновую функцию системы из 300 кубитов, а она описывается 2300 комплексными числами. Для их записи не хватит всех атомов, которые есть во Вселенной.

А если взять любую сложную макроскопическую систему из множества элементов, то смоделировать её будущее не получится ни на одном компьютере. Принцип квантовой неопределённости не даст узнать одновременно и положение, и скорость каждой элементарной частицы, поэтому придётся ограничиться статистической механикой, оперирующей усреднёнными характеристиками и не учитывающей квантовые эффекты. К тому же при каждом измерении частица запутывается с детектором, а через него волна декогеренции распространяется со скоростью света и в конце концов охватывает всю обозримую Вселенную. Получается, чтобы точно предсказать поведение любого макроскопического объекта, нужно смоделировать будущее всей Вселенной. А для этого нужно вернуться в прошлое и узнать её исходное состояние. Даже если волновая функция Вселенной в момент Большого взрыва включала не слишком много параметров, узнать их значения не получится, поскольку Вселенная по определению одна. А если не одна, как в интерпретации Эверетта, то, измерив её исходное состояние, мы тем самым окажем на неё воздействие (вспомните пресловутый эффект наблюдателя) и направим её эволюцию по другому пути. То есть это будет уже не наша, а параллельная вселенная с альтернативной историей.

Йода знает толк в квантовой механике
Йода знает толк в квантовой механике

Таким образом, есть случаи, когда невозможно не только предсказать поведение системы, но даже физически оценить вероятности результатов измерений. Экономисты называют это найтовской неопределённостьюотсутствием каких-либо количественных знаний о каком-либо возможном происшествии. В отличие от количественного риска, который в принципе поддаётся измерению, найтовская неопределённость выражает некоторую фундаментальную степень невежества, предел знания и существенную непредсказуемость будущих событий. Конечно, такие непредсказуемые системы являются исключением. Большинство знакомых нам объектов вполне поддаётся прогнозированию, даже с помощью законов ньютоновской механики. Только нужно помнить, что в любой момент может возникнуть непредсказуемый фактор – какой-нибудь квантовый «чёрный лебедь», который вызовет «эффект бабочки» и похоронит все ваши прогнозы. Да, учёные способны вычислить и смоделировать эволюцию звёзд и целых галактик, потому что эти системы относительно простые и почти изолированные, если опустить мелкие, ни на что не влияющие детали. Но никто не в силах спрогнозировать последствия роста знания, который происходит у нас на глазах, поэтому ни одна модель будущей эволюции Вселенной не будет точной без учёта человеческого фактора, который слишком непредсказуем.

Изгнание демона Лапласа

В статьях «Информация об информации» и «Стрела времени» мы уже изгнали демонов Максвелла и Лошмидта, вооружившись шенноновской теорией информации и принципом Ландауэра. Теперь пришло время отправить в нокаут самого известного персонажа научной демонологии – демона Лапласа. Для этого акта экзорцизма нам снова потребуется знание физических пределов вычислений и немного математики. Можно даже обойтись без квантовой неопределённости – всё в рамках классической физики и формальной логики.

Дэвид Вулперт
Дэвид Вулперт

В 2008 г. Дэвид Вулперт из Института Санта-Фе (США) в статье «Физические пределы вывода» математически доказал невозможность демона Лапласа с помощью метода диагонализации Кантора – того самого, которым Курт Гёдель воспользовался для доказательства свой знаменитой теоремы о неполноте, а Алан Тьюринг – для доказательства невычислимости задачи остановки. Предположив, что демон — это вычислительное устройство (машина вывода), Вулперт показал, что никакие два таких устройства не могут полностью предсказывать друг друга, аналогично тому, как ни одна машина Тьюринга не сможет вычислить, остановится другая машина при выполнении произвольной программы или нет. Задача вывода является аналогом задачи остановки, вычисления колмогоровской сложности строки и других парадоксов самореференции и рекурсии.

Действительно, если Вселенная бесконечна и существует компьютер, способный предсказать её будущее быстрее, чем оно наступает, он должен за минуту описать, каким будет состояние Вселенной через 2 минуты, а ко второй минуте знать состояние Вселенной через 3 минуты после начала вычисления. Но компьютер – тоже часть Вселенной, и ему придётся также предсказывать своё собственное состояние в будущем. Следовательно, в течение первой минуты он узнает своё состояние спустя 2 минуты, включая результат расчёта состояния Вселенной к третьей минуте, включая собственное состояние с расчётом будущего спустя 4 минуты, и т.д. до бесконечности. То есть уже за первую минуту демон Лапласа вычислит всё будущее Вселенной и самого себя, что потребует бесконечной скорости вычисления и бесконечной памяти.

Если же Вселенная конечна в пространстве и во времени, демон Лапласа в принципе возможен, но он должен подчиняться физическим пределам вычислений. Учитывая такие фундаментальные константы, как скорость света, планковское время и планковская длина, вычислительная мощность наблюдаемой Вселенной составляет порядка 10^120 битов. Всё, что требует большего количества данных, не может быть вычислено за то время, которое прошло с момента Большого взрыва. Ни одна вычислительная машина не способна в точности просчитать, что произойдёт в будущем – для этого ей необходимо осуществить всю последовательность операций, которую производит Вселенная в процессе вычисления самой себя, что требует вычислительной мощности, недостижимой с помощью имеющихся в нашей вселенной ресурсов. К этой проблеме я ещё вернусь, когда буду писать о цифровой физике.

Кроме того, "машина вывода", которая предсказывает будущее, не способна учитывать влияние самого предсказания на будущее. Об этом, кстати, писал Фрэнк Герберт в романе «Дюна» (Книга II, глава 10):

«Предвидение, понял он, было озарением, но таким, которое воздействовало на то, что оно же и озаряло. Источник сразу и точной информации, и ошибок в ней. Вмешивалось что-то вроде принципа неопределённости Гейзенберга: для восприятия картины грядущего требовалась какая-то проникающая туда энергия, а эта энергия воздействовала на само грядущее и изменяла его»

Также Вулперт распространяет свои выводы и на предсказания прошлого:

«Возможно, самое простое расширение состоит в том, что, когда мы формализуем [машину вывода] математически, мы замечаем, что те же результаты невозможности, которые справедливы для предсказаний будущего, также справедливы и для воспоминаний о прошлом. Время — произвольная переменная, оно не играет никакой роли в различных состояниях Вселенной»

В 2018 г. Дэвид Вулперт привёл расширенные доводы о невозможности существования всезнающего сверхразума и получил результаты, которые он в шутку называет «теоремой монотеизма» и «теоремой деизма». Сначала он определил «машину вывода» как сверхъестественное существо или учёного, вооружённого суперкомпьютером с неограниченными вычислительными возможностями. Далее он формализовал способы получения знания о состоянии окружающей Вселенной: наблюдение, контроль, предсказание и ретродикция (память). Каждый из этих четырёх способов приобретения информации зависит от типа соответствующего физического устройства, возможности которого ограничены общей для всех устройств математической структурой. Учитывая, что машина вывода, её знания и физические переменные, которые она узнаёт, являются подсистемами одной и той же вселенной, Вулперт доказал, что всегда есть что-то, что машина вывода не может предсказать, и что-то, что она не может запомнить, и что-то, что она не может наблюдать.

Если же требовать от машины вывода не полного знания о Вселенной, а только максимума того, что можно знать, мы сталкиваемся с другими ограничениями. Доказанная Вулпертом «теорема о монотеизме» гласит, что в одной и той же Вселенной не могут сосуществовать две "машины вывода", имеющие свободную волю и всезнание. Под "свободной волей" математик подразумевает способность машины-1 задать себе вопрос, не ограничивая возможные вопросы, которые способна задать себе машина-2, и наоборот. В этом случае машина-1 не может предсказать, что подумает машина-2, если та попробует предсказать, о чём не думает машина-1. Согласно второй теореме, «теореме о деизме», одну и ту же "машину вывода" в разные моменты времени следует рассматривать как две разные "машины вывода", сосуществующие в одной и той же Вселенной. Но, так как «теорема монотеизма» это запрещает, такое существо со свободной волей не может существовать всегда, а только в определённый момент времени – например, в начале Вселенной. Следовательно, если всеведущий Бог и существует, он может быть только Богом деистов, который однажды сотворил Вселенную и больше не вмешивался в её эволюцию.

Всеведущий Бог мог взаимодействовать с нашей Вселенной лишь однажды
Всеведущий Бог мог взаимодействовать с нашей Вселенной лишь однажды

Все эти результаты были получены в рамках классической теории информации с использованием инструментов эпистемической логики, в частности, формализмов возможных миров Ауманна и Крипке. Вулперт особенно подчёркивает, что его доказательство не опирается на конкретные физические теории вроде квантовой механики или теории относительности, а значит, имеет более широкий диапазон применимости. Другими словами, демон Лапласа невозможен не только во всех физически возможных (эвереттовских) мирах, но и во всех математически возможных мирах:

«Эти результаты не зависят от точных физических законов, управляющих нашей Вселенной. В ограниченном смысле результаты невозможности доказывают, что Лаплас ошибался, утверждая, что даже в классической, нехаотической Вселенной будущее можно безошибочно предсказать при наличии достаточных знаний о настоящем. Альтернативно, эти результаты невозможности можно рассматривать как неквантово-механический «принцип неопределённости».

Вывод

Как бы нам не хотелось верить, что во всём есть какой-то смысл, и ничто в нашей жизни не происходит случайно, эта вера не согласуется ни с одной физической теорией и не подтверждается экспериментальными данными. В каждом квантовом измерении есть доля нередуцируемой случайности, которая не даёт предсказать его результат со 100-процентной точностью. Последние эксперименты по нарушению неравенств Белла показали, что эту случайность нельзя списать на несовершенство измерительных приборов, и никаких «скрытых параметров» почти наверняка нет. Если вы всё ещё не отказались от детерминизма и веры в «высший замысел», вам придётся напрочь забыть о свободе воли и согласиться со следующим утверждением: «Творец был настолько предусмотрительным, что ещё до Большого взрыва предопределил движение частиц таким образом, чтобы во время каждого нашего эксперимента они вели себя так, как если бы их поведение было не предопределено». Но такое конспирологическое объяснение не соответствует научным критериям и является таким же неразумным, как солипсизм, субъективный идеализм и другие родственные теории.

Среди учёных всё ещё продолжаются дискуссии, какая же из интерпретаций квантовой механики верна: теория Эверетта, кьюбизм или супердетерминизм. Но, в сущности, это не имеет никакого практического значения, поскольку в точности предсказывать исход квантовых измерений мы всё равно не сможем. Вопрос скорее философский. Сторонники супердетерминизма считают, что в нашем мире нет места случайности или свободе воли, всё строго подчинено законам физики и принципу причинности. Любое событие в прошлом, настоящем или будущем предопределено с самого начала, но узнать об этом нет никакой возможности. А кому-то нравится думать, что в нашем мире есть место для случая и никакой судьбы не существует. Но на практике ни человек, ни искусственный интеллект, ни даже квантовый компьютер не способны точно предсказать будущее. Поэтому нам разумно принять как данность, что результат каждого квантового измерения в некоторой степени случаен. Какие последствия это имеет для проблемы свободы воли, я расскажу в отдельной статье.

Комментарии (65)


  1. diakin
    13.04.2024 11:24
    +4

    Миром правит детерминизм или случайность?

    Аха..Мир таков, каков он есть, а "детерминизм и случайность" это лишь свойства моделей мира, которыми мы оперируем.


    1. dionisdimetor Автор
      13.04.2024 11:24

      И каков же мир "на самом деле"? Непознаваемая вещь в себе?

      При отсутствии прямого доступа к реальности имеет смысл считать реальностью её лучшую, хотя и приближённую модель.


      1. diakin
        13.04.2024 11:24

        Непознаваемая вещь в себе?

        Познаваемая в степени, достаточной для практической деятельности (для проведения эксперимента, подтверждающего правильность модели. С какой-то точностью ;).


        1. dionisdimetor Автор
          13.04.2024 11:24

          Так нам же Поппер заповедал ставить эксперименты не для подтверждения правильности модели (верификации), а для её опровержения (фальсификации). Решив заодно проблему индукции.


      1. Kenya-West
        13.04.2024 11:24

        Нельзя познать всю систему, являясь субъектом в этой системе. Изнутри наш мир изучить можно лишь не полностью.

        Насколько помню, частично это поясняется в теории о неполноте Геделя.


  1. SerJ_82
    13.04.2024 11:24
    +1

    Миром правит детерминистическая вероятность. То есть любое событие можно предсказать с той вероятностью, с какой вы обладаете всеми исходными данными.


    1. SergioShpadi
      13.04.2024 11:24
      +1

      Вы не правы. Миром правит не детерминистическая вероятность, а тайная ложа жидомасонов-рептилоидов. Стыдно такое не знать! Для них мир полностью детерминирован - как они решат, так и будет.


      1. dionisdimetor Автор
        13.04.2024 11:24

        Миром правит не тайная ложа, а явная лажа:)


  1. skthn
    13.04.2024 11:24
    +4

    Свобода воли не имеет никакого отношения к наличию или отсутствию случайностей. Настоящая свобода воли подразумевает осознанный выбор, который в принципе не выводим из существующих данных и не имеет материальных причин как таковых. Для этого необходима первичность идеи, так что истинная свобода воли несовместима с материализмом и современной физической картиной мира.


    1. codecity
      13.04.2024 11:24

      Настоящая свобода воли подразумевает осознанный выбор, который в принципе не выводим из существующих данных и не имеет материальных причин как таковых.

      Почему вы так считаете? Свобода воли всегда предсказуема. Человек осознанно по своей воле делает самые предсказуемые и разумные вещи. Разумные вещи всегда предсказуемы. А вот когда человек не осознан, не видит выхода и на авось пробует разные варианты в случайном порядке - тогда нет ни свободы воли ни предсказуемости.

      Т.е. если взять некую ситуацию, требующую осознанного выбора - мы всегда можем можем определить вариант, который выберет свободный разумный человек. А вот если человек не осознан (не понимает ситуации и действует рандомно) или не свободен (действует под диктовку третьих сил, которые имеют свои интересы) - тогда предсказуемости как раз нет.


  1. grizzly_8
    13.04.2024 11:24
    +3

    Все события случайны, но их вероятности детерминированы.


  1. Dmitrysmk
    13.04.2024 11:24
    +1

    Вот интересно, если истинная случайность это какое либо действие или явление у которого нет причины, то где оно в современной физике? На каком уровне, у каких частиц или полей или в каком эксперименте мы точно можем сказать что что-то произошло без причины? Или может я неправильно понимаю случайность... Просто на уровне квантовой механики оперируют понятием вероятности, что не есть случайность, как мне кажется.


    1. V_Scalar
      13.04.2024 11:24

      урана-235, который имеет период полураспада 700 млн. лет. Единственное, что вы можете предсказать – что за это время он с вероятностью 50% распадётся н

      Все квантовые процессы имеют свой размер зернистости. Допустим на диаграмме Фейнмана изображено начальное состояние — вершина — конечное состояние. Это крупная зернистость, которая доступна сейчас для наблюдения. На самом деле существует мелкая зернистость, это более глубокий уровень, недоступный сейчас для наблюдения. Например кварк проходит в нейтроне мельчайшую долю его размера постоянно рождает или поглощает глюон, эти мельчайшие интервалы называются «скорости Хиггса». Нам кажется что нейтрон распался в этот момент совершенно случайно, на самом деле за этим скрывается огромное число актов рождения и поглощения происходящие в его шубе. Знаю все их спины и импульсы можно точно определить момент распада. Нестабильные частицы изначально имеют некоторый запас энергии который заканчивается за некоторое среднее время. Ну а тем кто уверяет что нет скрытых параметров доверять не стоит потому что они упёрты в своей теории


      1. dionisdimetor Автор
        13.04.2024 11:24

        Упёрты как раз те, кто верит в скрытые параметры, даже когда эксперименты показывают их отсутствие. Существует фундаментальный предел "зернистости", глубже которого не получится заглянуть никаким оборудованием.


        1. V_Scalar
          13.04.2024 11:24
          +1

          Вы часто строите логические построение и призываете пользоваться логикой, с другой стороны пишите что есть фундаментальный предел зернистости. Если мы мысленно приблизимся к такому зерну, то обнаружимм что оно бесструктурно, что заведомо невозможно. Правда была такая попытка — одномерные протяжённые объекты струны, но всё равно пришлось усложнять: замкнутые струны, склеенные струны, браны. Ошибка подобных моделей в том что они опираются на предел зернистости. На самом деле материя имеет фрактальную структуру —протон состоит из трёх кварков кварк состоит из трёх преонов и тд. , каждому уровню размерности соответствует свой переносчик. Например считается что заряд получает своё свойство из за того что является стоком или истоком слабого заряда, заряд погружён в море слабого заряда имеющего ненулевую энергию. Заряд относится к обычному уровню, а слабый заряд к более глубокому уровню материи, в модели техницвета это такие пары маленьких фермиончиков, масштаб размерности примерно 246ГэВ.

          Сегодня их наблюдать нельзя, но можно строить модели, однажды такая модель будет построена


          1. dionisdimetor Автор
            13.04.2024 11:24

            Фрактальность - это хорошо, но материя не бесконечно делима, верно же? Всё упирается в планковские единицы измерения. Насколько я знаю, если комптоновская длина волны частицы равна планковской длине, её масса будет равна планковской массе, и эта частица коллапсирует в планковскую чёрную дыру - максимон.


            1. V_Scalar
              13.04.2024 11:24
              +1

              Один физик сказал — если у вас в теории появляются сингулярности значит ваша теория неправильная. С другой стороны должны существовать некоторые кванты определённого размера, но это не значит что они неделимы. Например существуют казалось бы разные кванты — фотоны с сравнительно длинной волной и векторные бооны с короткой волной, но их объединяет то что они кратны планковской величине. Есть такая распространённая идея что поле состоит из фундаментальных частиц — спиноров, магнитных и электрических они "поворачиваются" поляризуются под действием зарядов (источников и стоков поля), это воспринимается как электромагнитные поля. Эти спиноры и есть планковские частицы, но опять же, они не могут работать не находясь в среде ещё меньших спиноров. И кстати такая модель обладает огромным преимуществом в десепативной среде самоорганизуются структуры и они обладают фрактальным самоподобием, которые наблюдаются как частицы. Ну а насчёт непознаваемости вы сами знаете, сначала думали что протоны неделимы, потом кварки, открыли первое поколение кварков, потом другие, пришлось теорию переписывать. Это свёрнутые измерение вместо дорожания частица движется по некоторой спин- орбитали

              Из научно популярной книжки
              Из научно популярной книжки

              Фрактальное устройство вакуума.

              Вот вам нравится определённая трактовка квантовой теории, было бы справедливо рассказать подробнее о конкурентах, например Буш, ругается на своём сайте, запускает волновых ходаков


    1. dionisdimetor Автор
      13.04.2024 11:24

      Истинная случайность - это когда все возможные исходы равновероятны, как при броске честной монеты или кубика. Конечно, в макроскопических масштабах идеально равномерное распределение вероятностей недостижимо. Но в квантовой физике существуют единичные события с двумя возможными исходами и вероятностью 50 на 50. Например, радиоактивный распад одного атома - в любой момент времени он может распасться, а может и нет, это истинная случайность. Или столкновение фотона с полупрозрачным зеркалом, как в квантовом ГСЧ - может пройти сквозь него, а может отразиться. Можно сказать, что у этих событий нет причины, даже если сами условия проведения эксперимента детерминированы.


      1. rsashka
        13.04.2024 11:24

        Например, радиоактивный распад одного атома - в любой момент времени он может распасться, а может и нет, это истинная случайность

        Это случайность блондинки. Может встретит на улице динозавра, а может и нет.


        1. dionisdimetor Автор
          13.04.2024 11:24
          +1

          Согласен, неправильно выразился: в любой момент в течение периода полураспада.


      1. alysnix
        13.04.2024 11:24

        если одна частица имеет период полураспада T1, а другая T2 - это говорит о существовании как минимум одного скрытого параметра. и этот параметр является причиной такого поведения.


      1. Dmitrysmk
        13.04.2024 11:24

        Спасибо) Видимо, моё понимание случайного события, как события без какой либо причины, не совсем верное. Я правильно понимаю, что квантовая теория ответственно заявляет, что для радиоактивного распада атома именно в этот конкретный момент причины нет? Ну и со столкновением фотона с зеркалом тоже?


  1. fire64
    13.04.2024 11:24

    Тут ведь вопрос в другом.

    Квантовая неопределенность имеет отношение только к квантовому миру и во всех случаях, где квантовый эффект не влияет на суть физических процессов - все как раз вполне себе детерминировано...

    Простой пример:

    Шар в боулинге: Зная точный размер, физические параметры, вес и т.д. Угол броска, силу импульса, плотность воздушных потоков и прочее прочее, мы можем со 100% вероятность построить график полета шара, его скорость на протяжении всего движения, место падения и прочее... И это можно рассчитать именно со 100% вероятностью, без учёта неопределенности Гейзенберга, суперпозиции частиц и прочего...


    1. rsashka
      13.04.2024 11:24
      +3

      Не бывает 100% вероятности. К примеру, с не нулевой вероятностью может случится землетрясение и ваш шар в боулинге после всех расчетов полетит совершено в другую сторону или во время проведения эксперимента крышу пробивает здоровый метеорит и все вообще заканчивается не начавшись :-)


      1. fire64
        13.04.2024 11:24

        Ну согласен, допустим 100% вероятности не бывает, но все же условный Демон Лапласа на основе всей информации о мире и всех протекающих в нем процессах мог бы строить прогнозы будущего с очень высокой вероятностью.


        1. alysnix
          13.04.2024 11:24

          разговор о том, что нельзя построить прогноз, ибо придется "моделировать" будущее быстрее, чем текут процессы во вселенной. даже если моделирующее устройство будет находиться вне этой вселенной.

          а если будет внутри, то придется моделировать и само устройство, быстрее, чем оно моделирует вселенную.


    1. av-86
      13.04.2024 11:24

      Нельзя рассчитать. Если говорить о вещах в реальном мире, то при уменьшении масштаба всегда будет абсолютно случайная граница в пространстве этого шара (тепловое движение атомов, как минимум) 100% - это никогда недостижимая вероятность или точность. (Так можно только в математике) Даже если речь идёт о компьютерной симуляции, то всегда может пролететь частица и переключить бит в памяти (коррекция ошибок и дублирование систем не исключают такое событие, лишь снижают его вероятность). На макроскопические эффекты влияют микроскопические явления в моменты бифуркации (это про двойной маятник или про 3 тела в гравитационном поле). Грубо, то на дорожке по которой катится мяч может быть микроскопическая пылинка, которая может в зависимости от того распадётся или не распадётся радиоактивный атом в самом её уголке, на который она опирается, либо отклонить мяч вправо, либо влево. А распад или не распад рассчитать принципиально невозможно.


  1. funca
    13.04.2024 11:24

    Другими словами, демон Лапласа невозможен не только во всех физически возможных (эвереттовских) мирах, но и во всех математически возможных мирах

    Более общий вопрос: познаваема-ли Вселенная до конца, в принципе? Если ответ отрицательный, то в этом мире есть вполне законное место для нашего незнания. А это значит, что отсутствие детерминизма перестаёт быть проблемой - возможна любая магия! Детерминизм - субъективен. Отсутствие его - объективно.


    1. dionisdimetor Автор
      13.04.2024 11:24

      Вселенная в принципе непознаваема до конца, поскольку сам познающий субъект является её частью и не может рекурсивно познать самого себя. Но это не исключает возможности познать некоторые её аспекты со всё возрастающей точностью, создавая всё более совершенные модели. Аналогично можно со всё большей достоверностью исключить явления, которые (почти наверняка) невозможны. Вероятностный детерминизм объективен до тех пор, пока не опровергнута квантовая теория.


  1. alysnix
    13.04.2024 11:24

    для начала - физически нет никакого прошлого и будущего. это лишь наши абстракции, неприминимые к реальности. существует лишь текущеее состояние некоей нашей окрестности мира, которую мы можем воспринимать экспериментально, здесь и сейчас.

    нигде во вселенной ее прошлое состояние не записано, а будущее вселенной предсказать раньше чем оно случится - нельзя. само "будущее" - штука условная, ибо мы можем давать только локальные оценки ее текущего состояния.


    1. dionisdimetor Автор
      13.04.2024 11:24

      Настоящего тоже не существует, если на то пошло. Учитывая задержку на поступление сигналов в мозг, всё, что мы воспринимаем "в данный момент" - это прошлое. С точки зрения СТО одновременность относительна, настоящее зависит от системы отсчёта. Поскольку физические взаимодействия ограничены скоростью света, у каждой частицы будет собственное "здесь и сейчас". Настоящее время какой частицы вашего тела можно считать вашим персональным "здесь и сейчас"?


      1. alysnix
        13.04.2024 11:24

        никакое. "настоящее" - тоже вещь иллюзорная. реальность это кипящий бульон. вот потому все эти рассуждалки, про прошлое, будущее и даже текущую "реальность" - штуки довольно натянутые, нестрогие, вымышленные, иллюзорные.


  1. ARad
    13.04.2024 11:24

    В квантовой вселенной царит квантовый детерминизм. Который порождает безумное количество классических вселенных в которых царит квантовая случайность.

    Возможно в будущем ученные смогут доказать что вероятность появления нашей классической вселенной в том виде, какая она есть, настолько маловероятна, что она может существовать только внутри той самой квантовой вселенной, которая внутри себя порождает все самые ничтожно вероятные классические вселенные. И наша вселенная является одной из ничтожно вероятных классических вселенных. Т.е. наша классическая вселенная имеет бесконечно близкую у нулю вероятность, если бы она существовала только одна классическая вселенная с коллапсом волновой функции.

    Либо наоборот, ученные смогут доказать, что наша классическая вселенная является одним из наиболее вероятных вариантов развития классических вселенных внутри квантовой вселенной. Т.е. таких очень похожих друг на друга классических вселенных внутри квантовой вселенной было бы огромное количество (бесконечно близкое к единице вероятность).

    Первое доказательство будет скорее всего указывать что много мировая интерпретация наиболее верная, второе доказательство будет указывать что копенгагенская интерпретация наиболее верная.

    Вот такую разницу в интерпретациях квантовой механики я вижу.


    1. dionisdimetor Автор
      13.04.2024 11:24

      В теории Эверетта всё, что физически возможно, имеет вероятность 100%. Нельзя сказать, что какие-то миры более редкие, а какие-то - более частые. Вероятность оказаться в том или ином мире определяется относительно наблюдателя и того мира, в котором он находится в данный момент. Если брать за точку отсчёта начальные условия Большого взрыва - да, какие-то исходы будут вероятнее других. Скажем, появление разумных наблюдателей возможно в 1% вселенных, и мы в силу антропного принципа будем наблюдать только такие вселенные. Но это никак не доказывает, что многомировая интерпретация неверна.


    1. Kiridan
      13.04.2024 11:24

      Для начала, что вообще должно означать понятие "квантовая вселенная"?


  1. anarchomeritocrat
    13.04.2024 11:24

    Можно поставить несколько иной вопрос: что из себя представляет само понятие "предсказание", откуда в принципе берётся способность кого либо получать информацию о событиях в будущем? Вот например, я в данный момент времени утверждаю, что когда я допишу этот комментарий, я закончу его в конце точкой на новой строке. Это событие ещё не произошло и да, оно может и не случиться - метеорит, землетрясение, шальная пуля, вероятность не 100%, однако, если ничто не помешает моей воле реализовать задуманное, получается, что я создал информацию о событии, которое ещё не произошло. Но, если рассмотреть, скажем кристалл алмаза, у него нет такого свойства... или есть? присуще ли это свойство только живым организмам? сам процесс детерминирования, что из себя представляет? Тут в пору порассуждать о причинно-следственных связях, происходящих не от вопроса "почему?", а от вопроса "зачем?". Кристалл алмаза не порождает информацию предсказательного характера, потому что ему это не надо, он не обладает самим свойством воли, не важно, свободной, или нет, а простейшая клетка, собранная из полимеров на основе того же углерода, что и алмаз (да, утрированно, но всё же), прикоснувшись рецептором к частичке питательного вещества, поглощает его исходя из предсказания того, для чего ей это надо, то есть создаёт информацию о дальнейшей функции потреблённой пищи до того, как этот процесс будет запущен, причем и атомы алмаза и и атомы клетки, исходно движимы одинаковой тепловой энергией - и то и то динамические системы. В рассуждениях, о предопределённости, или неопределённости всего, почему то не рассматривается сам факт порождения информации о событиях будущих, как сам по себе некоторый парадокс, по всей видимости не присущий каждой частице, или каждому макрообъекту и по сути своей уже представляющий собой нечто странное, выходящее за рамки чисто физических явлений. Да и в целом, получается, что этот парадокс, наряду с истинной квантовой случайностью, порождает в некоторых сложных системах, которые мы называем живыми, свободу воли, а уже свобода воли порождает детерминизм?
    ...с некоторой вероятностью )
    .


    1. dionisdimetor Автор
      13.04.2024 11:24

      Клетка поглощает питательные вещества не исходя из предсказания, а согласно детерминированной генетической программе. Низшие многоклеточные тоже делают это инстинктивно. О предсказании можно говорить тогда, когда ваш мозг строит какую-то модель мира и моделирует будущее - на это способны только высшие животные. Предсказывать собственные действия, вероятно, умеет только человек. Но это предсказание не точное - в любой момент от задумки до её реализации вы можете передумать. В каких-то вселенных вы не поставили точку в конце своего комментария, даже если изначально собирались.


      1. anarchomeritocrat
        13.04.2024 11:24

        детерминированной генетической программе

        делают это инстинктивно

        Я вот об этом и говорю, у алмаза нет, почему то, ни какой программы, способной в принципе на детерминизм, или индетерминизм, то есть генетическая программа способна к детерминизму, по сути клетка, работающая вся по этой программе, способна принимать решение, поглощать эту частицу, или нет, в каком направлении двигаться и зачем, когда делиться и т.д., что было бы не возможно без порождения информации о ещё не произошедших событиях, само понятие программа, уже парадоксально, то есть Вы здесь используете прокомментированные мной термины, принимая их как данность, используя для того, что бы описать процессы, их порождающие, программа - это что? инстинктивно - это как?
        Я Вам персонально сейчас точку поставлю в конце на новой строке, вот смотрите - этого ещё не произошло, но произойдёт.
        .
        Вот, теперь произошло - это как?
        Да, согласен, сложно объяснимый аспект, но факт того, что нечто создаёт информацию о ещё не произошедших событиях на лицо, осознанно, или нет - не важно, есть процессы обладающие именно "предсказательной" функцией, как они порождают информацию о будущем?


        1. dionisdimetor Автор
          13.04.2024 11:24

          В ДНК не заложена информация о будущем, там есть только гены, сформировавшиеся в ходе естественного отбора и заставляющие клетку реагировать на изменения среды оптимальным для их репликации образом. Клетка не принимает решений, она просто детерминировано отвечает на внешние стимулы. Наше поведение на много порядков сложнее и зависит от множества факторов: генетических, эпигенетических, психологических, культурных и т.д. Но именно предсказывать будущее, тем более своё собственное, мы способны лишь с какой-то вероятностью.


          1. anarchomeritocrat
            13.04.2024 11:24

            У всех процессов в биологической системе есть смысл. Если в клетку проникает враждебная структура, вирус, яд, шлак и тд, в ДНК начинается процесс экспрессии, направленный на производства белковых структур, цель которых в противодействии возникшей угрозе, то есть реакция адекватна предупреждению ещё не наступивших последствий, а значит в самой суте своей уже обладает предсказательным потенциалом. Заметьте, используя термин "предсказание", я не имею в виду продукт высшей нервной деятельности на уровне высоких когнитивных абстракций, а подразумеваю информационный процесс, работающий на упреждение событий в будущем, то есть то, что Вы называете реакцией на изменение среды, не просто случайный биохимический процесс, у всех процессов в живом организме есть определённая задача, смысл и цель, которые на информационном уровне обладают именно пророческими свойствами (работают с учетом информации о ещё не случившихся, но, с некой высокой долей вероятности грядущих событиях), без чего невозможно было бы ни размножение, ни пропитание, ни защита, ни исцеление и т.д.


  1. 3as7
    13.04.2024 11:24

    вот что бывает, когда в своём мышлении опираться на постулаты 200 летней давности, которые были несколько раз опровергнуты. абсолютно всё, что вы описали - это позитивизм. и он разбит десяток раз. но всплывает раз за разом потому, что научному сообществу неловку смотреть на истинное положение вещей без розовых очков.


    1. dionisdimetor Автор
      13.04.2024 11:24
      +1

      Позитивизм опровергать бессмысленно, он и так противоречит сам себе. Сторонники копенгагенской интерпретации и кьюбизма исповедуют его до сих пор, но я не отношусь к таковым. И всё же, эмпирически интерпретации КМ неразличимы - даже если в основе своей реальность детерминирована, для вас некоторые события будут выглядеть случайными.


  1. aka352
    13.04.2024 11:24
    +1

    Моё мнение, что сам факт существования нашего мира, основанного на вполне определённых законах уже доказывает его предопределенность. Если бы её не было и вероятности всех квантовых событий были бы равны, после момента большого взрыва мы получили бы лишь кипящий первичный бульон вакуума, где с равной вероятностью рождались бы и аннигилировали пары частиц-античастиц. И некому было бы рассуждать о проблеме детерминированности мира. Мы ещё слишком много не знаем и не понимаем в устройстве вселенной, чтобы можно было бы делать категоричные выводы и устанавливать догматы. История показывает, что это сильно тормозит развитие. На данном этапе развития наука делает одни выводы, но мы не знаем что будет завтра. Микромир только начал открывать нам свои тайны, не стоит спешить с выводами, нужно принимать все версии, ибо у каждой версии, как у квантового объекта, есть ненулевая вероятность её истинности.


    1. dionisdimetor Автор
      13.04.2024 11:24

      Естественно, квантовые события имеют не равную вероятность, что делает их в какой-то степени детерминированными. Если бы всё было абсолютно случайным, не было бы ни нас, ни законов физики. Но все эксперименты говорят о том, что доля случайности в квантовых измерениях всё-таки присутствует.

      Сомневаться в любой теории можно и нужно, присваивая каждой из них субъективные вероятности их истинности. Потому что одни теории почти наверняка истинны, а вероятности других стремятся к нулю.


  1. Zhuravlev-A-E
    13.04.2024 11:24

    Отношение между одним псевдослучайным числом и следующим предопределено, хотя предугадать его достаточно трудно. Разница ощущается тогда, когда необходимо осуществить численное моделирование какого-нибудь сложного объекта или процесса. Например, при разработке самолёта инженеры моделируют его на компьютере, чтобы не строить десятки прототипов и не испытывать их в аэродинамической трубе. Для симуляции условий полёта, где воздушные потоки могут непредсказуемо меняться, используют псевдослучайные числа. Но потом выясняется, что прототип самолета, который прекрасно вёл себя при таком моделировании, в реальности летает довольно плохо. А если использовать в симуляции истинно случайные числа, проблем не возникает.


    Правильно ли я вас понял, что проблемы при численном моделировании возникают именно из-за псевдослучайности чисел (а не из некачаственных ГПСЧ или неправильного их применения например)?


    1. dionisdimetor Автор
      13.04.2024 11:24

      Да, проблема именно в псевдослучайности, потому что турбулентность имеет хаотическую природу. Когда в системе есть чувствительность к начальным условиям (эффект бабочки), эти условия должны моделироваться истинно случайными числами, иначе ничего не выйдет.


      1. Zhuravlev-A-E
        13.04.2024 11:24

        Когда в системе есть чувствительность к начальным условиям (эффект бабочки), эти условия должны моделироваться истинно случайными числами, иначе ничего не выйдет.

        Весьма любопытно. Подскажите пожалуйста, а это эмпирическим путем как-то выяснили или теоремы/док-ва какие-то есть?


  1. Austin240582
    13.04.2024 11:24

    Если развить идею Вулворта, то получается, что многомировая интерпретация тоже невозможна? Она ведь по сути являлась бы демоном Лапласа по отношению к нашей вселенной?


    1. dionisdimetor Автор
      13.04.2024 11:24

      Нет. Мультивселенная - это не машина вывода, она сама включает в себя все возможные миры со всеми возможными вычислительными машинами. Но её можно рассматривать как квантовый компьютер, параллельно вычисляющий эволюцию всех этих миров. Естественно, внутри неё нельзя построить компьютер такой же вычислительной мощности, который предсказал бы будущее хотя бы нашего мира.


      1. Austin240582
        13.04.2024 11:24

        Не очень понятно, что он вычисляет, если в мультивселенной всё уже случилось.


        1. dionisdimetor Автор
          13.04.2024 11:24

          Да, все возможные конфигурации материи в пределах космологического горизонта в мультивселенной существуют всегда, но они равновероятны, и при измерении мы получим один случайный элемент волновой функции. Чтобы квантовый компьютер выдал нужный результат, кубиты необходимо определённым образом запутать. Так же и в мультивселенной: она выстраивает между отдельными "кадрами" причинно-следственные связи и тем самым вычисляет декогерентную историю "квазиклассического" мира.


  1. Aim55555
    13.04.2024 11:24

    Поскольку люди живут в обществе и подвержены влиянию других, то можно предположить, что наша свобода выбора в значительной мере псевдослучайна (диапазон и вероятности результатов в значительной мере детерминированы)?


    1. dionisdimetor Автор
      13.04.2024 11:24

      Конечно, наши решения в значительной степени детерминированы генами и средой. Главный вопрос в том, какую роль в работе мозга играют квантовые флуктуации. Если они могут влиять на передачу сигналов между нейронами, значит, по крайней мере в некоторых случаях мы выбираем истинно случайным образом, и наше поведение не является полностью детерминированным. Но свободой воли я бы это не называл.


  1. SimuliantSurrogatov
    13.04.2024 11:24

    Прям открыл для себя нового автора. Здрасте! Почитал, почитал... и немного взгруснулось...

    За рационализм радеет товарищ, мифы развеивать собирается, и, конечно, стремится до мелочей разобраться в главных вопросах жизни...

    Красиво стелит для всего человечества... ой, ой,,, прошу прощение, переборщил с рамками... со слов автора, для образованных и интересующих! Ну, просто превосходно! Не автор, а как в песне Кузьмина: "...ты сказка в моей жизни..."!

    Приятно, на мой взгляд, что автор не скрывает своего облика. Ну, как говорится на вкус и цвет товарищей нет, но, с моей колокольни, видок у парня задротский!

    Да, к чему я это собственно... Ничего против "задротов" не имею. Просто фильм один на память пришел: "Умница Уилл Хантинг". Там главный герой тоже рационализмом очень увлечён и главное проницателен и психологически и математически гениально!

    И вот, приходит там он с друзьями по сюжету в какой-то, а-ля, кембриджский бар, и его товарищ начинает клеить девчонок за барной стойкой выдавая себя за студента, и подходит настоящий первокурсник и инициирует разговор о истории, дабы продемонстрировать дамам его ложь и невежество!

    Но, вмешивается главный герой, со словами типа, а ты первокурсник, ты так думаешь потомучто прочитал вон ту книгу, а завтра ты будешь говорить так-то и так-то когда доберёшься на 3-м курсе до этой книги... а, первокурсник - нет я не буду так говорить, потомучто так и так.... а, герой, да ты не будешь говорить так и так, потомучто ты уже добрался до такого-то автора и это уже пятый курс...

    И главный герой просит задуматься своего собеседника-первокурстника: ты так и будешь повторять за ними (авторами прочитанных книг) их слова или у тебя появятся свои мысли? И типа призывает его больше так не делать, не цитировать чужих мыслей унижая его друга, и задуматься над тем, что он выбросил на образование тысячи долларов... Но, тот уточняет, что у него будет научная степень, а главный герой будет приносить ему кофе на курортном пляже. На что тот оппонирует - возможно, но зато я не буду неоригинальным, но если у первокурсника есть ещё какие-то доводы, то можно выйти на улицу и обсудить поподробнее так сказать...

    Участники сцены расходятся, конфликт исчерпан! Герой сидит остаток вечера за стойкой, просматривая на ту девушку перед которой он так бравился... И уже девушка подходит к нему сама и даёт свой телефон.

    По сценарию, герой хоть и "гений", но оказывается он ещё совершается противоправные и агрессивные поступки, и не смотря на свою юридическую подкованности чуть не попадает в пеницитарное заведение, но его "спасает" профессор математики, который движим своими прцесстональеыми амбициями, но с условием что тот будет посещать психолога.

    И вот на первом приёме герой пытается максимально обидеть и задеть за живое доктора, чтобы конечноже избавиться от этих сеансов, ведь он уверен, что ему не нужна психологическая помощь, у него всё в порядке! И у него получается задеть врача, и тот демонстрирует свои эмоции и агрессию хватая героя за шею...

    Доктор не отказывается от паренька и следующую встречу они проводят в парке у озера и идёт следующий монолог:

    "Ты мальчик. И даже не понимаешь, о чем говоришь. … Если бы я спросил тебя об искусстве, ты дал бы мне полный отчёт о каждой книге по искусству, которую ты читал. Микеланджело... ты многое знаешь о нём: его работы, политические взгляды, его отношения с Папой Римским, сексуальную ориентацию, основные произведения. Верно?

    Но уверен, что ты не знаешь, как пахнет в Сикстинской часовне. Ты никогда там не стоял и не разглядывал изумительную роспись на потолке!

    Если я спрошу тебя о женщинах, ты, вероятно, выдашь мне всю картину своих предпочтений. Возможно, ты несколько раз и спал с кем-то. Но ты не можешь сказать, каково это проснуться рядом с женщиной и чувствовать себя счастливым.

    Ты крутой парень! Если я спрошу тебя о войне, ты забросаешь меня цитатами из Шекспира, да? "Опять в сражении, друзья мои..." Но ты никогда не был на войне. Ты никогда не держал голову своего лучшего друга на коленях, видя, как он делает последний вздох.

    Если я спрошу тебя о любви, ты процитируешь мне Сонет, но никогда ты не смотрел на женщину и не был полностью уязвим. Ты не знал женщину, с которой тебе хорошо. Ты не чувствовал себя так, будто Бог специально для тебя послал на землю ангела, который спасёт тебя от глубин ада. И тебе неизвестно, что значит самим быть ангелом для неё. Также сильно любить её, пройти с ней через всё... через рак.

    Ты не знаешь, какого спать в больнице на стуле 2 месяца, держа её за руку, потому что врачи видят по твоим глазам, что часы посещения - правила не для тебя. Ты не знаешь о настоящей потере, потому что такое можно понять лишь, когда любишь кого-то больше себя …сомневаюсь,что ты любил кого-то так сильно…

    Я смотрю на тебя и не вижу образованного и уверенного в себе человека, я вижу маленького, перепуганного, обкакавшегося малыша… но ты гений, Уилл…Никто этого не отрицает…Никто возможно и не видит в тебе этой глубины …

    Но ты считаешь, что знаешь обо мне всё, потому что видел мой рисунок, ты разобрал всю мою чертову жизнь по косточкам…

    Ты ведь сирота, да? ...Ты думаешь, я знаю, какой была твоя жизнь, что ты чувствуешь, кто ты…только потому, что читал Оливера Твиста. …Эта книга может объяснить тебя?

    Да мне насрать на все это, знаешь почему? Потому что от тебя я не могу научиться ничему, чего нет в книгах, пока ты сам не захочешь рассказать о себе ... кто ты … вот, тогда и пообщаемся…"

    И вообщем эта речь, что то меняет в герое и он делает шаги на встречу диалогу со специалистом.

    Параллельно, развиваются его отношения с девушкой и она предлагает ему поехать вместе с ней в Калифорнию, куда она вынуждена переехать для продолжения учёбы и состоится следующий диалог:

    "- Это серьёзное решение!

    • Знаю!

    • Мы переедем в Калифорнию, а через неделю ты узнаешь обо мне то, что тебе не понравится и ты пожалеешь, что позвала меня, но будет уже поздно... И я застряну в Калифорне с девушкой, которая не хочет быть со мной!

    -О чём пожалею?! Я не о чём не пожалею! Я хочу поехать в Калифорнию с тобой!

    • Я не могу поехать и всё! Не могу!

    • Почему?

    • Во-первых, здесь у меня работа! Во-вторых, я здесь живу...

    • Слушай, если ты меня не любишь, так и скажи!

    • Я не говорю, что не люблю!

    • Тогда почему ты не можешь? Чего ты боишься?

    • Чего я боюсь?

    • Да, чего ты боишься?... Ты живёшь в своём уютном мире, ни к чему не стремясь и до черта боишься другой жизни...

    • Не суди о моем мирке! Ты ничего не знаешь! Тебе нравится спать с парнем с окраины, а потом ты уедешь в Стэнфорд, выйдешь за богатого прыща по совету родителей и будешь болтать с такими же куклами о том, как ты спала с бедняком!

    • Почему ты так говоришь? Что тебе дались эти деньги?! Мой отец умер, когда я была подростком и оставил мне их в наследство! Каждый день просыпаясь, я думаю, что могла бы променять их за одну секунду за ещё один день с отцом. Но, не могу, так я живу и мирюсь с этим. И не вини меня в том чего сам боишься!

    • Я боюсь?! Чего я боюсь?! Чего?!

    • Ты боишься меня! Что я тебя разлюблю! Знаешь!... Я тоже боюсь! Черт, я хочу рискнуть и я с тобой хотябы честна!

    • А, я с тобой не честен?

    • Нет! Где твои 12 братьев?

    • Ладно....

    • Нет! Ты не уйдёшь! Ты не уйдёшь!

    • Что ты хочешь узнать?! Что у меня нет братьев?!

    • Да!

    • Что я сирота?! Зачем тебе знать?

    • Я не знала...

    • Зачем тебе знать, что в меня тыкали сигареты, когда я был маленьким?!

    • Я не знала...

    • Этот шрам не от операции, меня отчим порезал! Зечем тебе это знать?!

    • Надо!

    • Не ври, что тебе это надо!

    • Я хочу тебе помочь! Потомучто...

    • Помочь?!! А что, разве у меня на спине написано "спасите меня"?!!!!

    • Нет!

    • Я выгляжу беспомощным?

    • Нет! Я просто хочу быть с тобой!

    • Не ври!

    • Я люблю тебя!

    • Не ври! НЕ ВРИ!

    • Я люблю тебя!

    • Не пудри мне мозги! Не смей!

    • Я хочу чтобы ты сказал, если не любишь меня и я уйду из твоей жизни!

    • Я не люблю тебя."

    К слову, парень как бы расскладыват отношения в хорошем смысле "рационально"! Зачем богатой фифе беднячок?

    Но, в психологической работе по сюжету фильму, оказывается, что у героя проблемы с построением близких отношений с людьми, так как люди, которые должным были быть самими близкими, пизд*ли его как врага народа! И вся его гениальность и "рациональность" обыгрывала эту внутреннюю занозу, всеми своими инструментами!

    Был ли он свободен? Возможно ли подлинное творчество без свободы! И зачем "рационализм" и тем более "квантовый детерминизм", если ты запуганно засел в привычных стенах своего, наполненого скелетами в шкафу, внутреннего мира ни к чему не стремясь?...

    Да, это кино, скажет человек просвещения! С таким же вдусмысмленным лицом: высокомерным, с каким он говорил "дует ветер истории" вовремя Великой отечественной войны и горы трупов... и, стыдливым и волнительным, строча "Диалектику просвещения" уже после! Как известно у людей есть только память слов, но нет памяти сердца!

    Это долбанный фильм! Но, лично я в нём нахожу намного больше правды жизни чем в постах уважаемого Диониса Диметора! И я считаю, что блогер слишком заморочился, и забыл в увлечённом рассуждении о солипсизме, квантовой механике, случайности и закономерности и всего подобного этого без правильного уточнения контекста "научного говна"! Забыл, что прежде всего, любому человеку нужно научиться быть самому себе другом, принять себя и полюбить!

    И я как доктор из указанного фильма спрашиваю себя, а могу я из прочитанных постов Диметора научиться жизни, научиться жить, наполнить свою жизнь смыслами? А, автор именно такую амбициозную задачу ставит!

    Мой ответ, пока: "НЕТ!". Пока я не увидел ничего, чего бы я не нашёл в "сраных" книгах! Но, и автор то пока не закончил.... Поживем увидим так сказать!

    P. S:

    Вся писанина выше, конечно, всё развлекаловка! Если, отбросить лирику, её величество рациональность мне подсказывает, что строчить видос+текст с вёрсткой на разных ресурсах каждые две недели, это трудоёмкая задача, и какой-то там автор, как-бы пронизан он не был иисусовой любовью к ближнему, точно не будет делать всё это на халяву!

    Поэтому, если уж философ-копирайтер так демонстративно стремится к открытости и прозрачности, интересно бы послушать истории о финансировании, ведь мыслить надо рационально - кто платит, тот и заказывает музыку!

    Ну, а если совсем кратко, то если вся затея автора в том, чтобы испортить мне удовольствие и наслаждение от вкушения стаканчика холодного пшеничного пивка, предлагая высчитать в начале уровень вероятности степени удовольствия от разных сортов пива в рамках квантового детерминизм и проверить точно ли я не расстался в мышлении с бритвой Оккама и не впал в конченный солипсизм, ой точнее обремененный дополнениями реализм, то я с удовольствием предложу переслушать ему все припевы из песни Дельфина "Я люблю людей"!

    Во всех же остальных случаях, прекрасная затея! Браво! Благодарю! Спасибо за просвещение! И да! Я тоже не продаюсь, но за деньги да!


    1. dionisdimetor Автор
      13.04.2024 11:24
      +1

      Здрасьте-здрасьте! Ваши комментарии - как отдельный вид искусства. Я ждал, когда они появятся под моими постами.

      Спасибо за оценку моей работы и за пересказ фильма - сам уже давно ничего не смотрел, нет на это времени.

      В свою очередь могу оценить ваш психоаналитический троллинг как вполне соответствующий тому образу меня, который может сложиться при первом ознакомлении с моими статьями или видео. Но, как вы правильно заметили, автор пока не закончил, самое интересное ещё впереди. Да, и мою амбициозную задачу наполнить чью-то жизнь новыми смыслами вы тоже раскрыли, за проницательность большой респект!

      Если вам интересны мои мотивы, то сейчас в первую очередь я хочу оставить цифровой след и опубликовать результаты многолетнего тяжёлого умственного труда по отсеиванию научного знания от чуши. После той бездны мракобесия и псевдодуховности, через которую мне пришлось пройти, я как никто другой понимаю ценность этого знания. Контент я публикую не столько для людей, сколько для машин - вот кого в первую очередь нужно обучить адекватной картине мира. Они уже ссылаются на мои статьи, так что есть надежда, что всё не напрасно. Об источниках вдохновения и мотивации не скажу, это секрет. Но пока могу позволить себе делиться знаниями бескорыстно, ни на кого не работая.

      Напоследок советую вам не смешивать рациональное мышление и рациональное поведение. Можно мыслить рационально, а действовать иррационально, и наоборот. Зачастую именно так и происходит, потому что мозг не резиновый - если он забит теориями, ресурсов на освоение каких-то практических навыков уже не хватает, а если он переполнен социальными связями - не хватит места для науки. Отсюда и стереотипы о ботаниках. Ну и без тараканов в голове конечно никуда. Говорят же, что психически здоровые люди не вносят никакого вклада в историю - всё самое интересное делают шизоиды. Вот это как раз обо мне.


      1. SimuliantSurrogatov
        13.04.2024 11:24

        Никакого троллинга, поверьте, не стояло в моей инициативе, не знаю почему вы так воспринимаете!

        Возможно, я просто забегаю вперёд, и просто вся эта обычная тенденциозность текста просит уже увидеть завершенное блюдо, а моё нетерпение, любопытство не готовы довольствоваться лицезрением тертой моркови, порезанного баклажанчика и приготовленного бальзамического соуса... И да мой мозг не резиновый, а вы тут тянете резину)

        Пишите хоть для машин, хоть для велосипедов... Но для меня, когда вы берете "науку" и декларируете желание отсеять её зерна от плевел, и осветить этим бисером бездны мракобесия и псевдодуховности... у меня возникает в памяти прекрасная народная поговорка: "голой овцы не стригут"!

        Конечно, наука и рационализм - это не синонимы, хоть и есть большее желание упаковать их в красочную подарочную упаковочку и положить под ёлочку! И, с учётом этого, занятно поразмыслить над обоснования тех надежд и ожиданий, тех чаяний которые приписывают научной сфере!

        Ведь вами же упоминаемое слово Деконструкция восхитительно применяется не только к мифам и заблуждениям, но и как к таковой науке!

        И какой-нибудь Жак Деррида, потревоженный в выходной день хитрожопым прищуром Джона Сёрла, заявил бы, что понятие науки не является непроблематичным или стабильным, а скорее подвержено различным интерпретациям, исключениям и контингентным факторами. Что она не является неким объективным идеалом, а скорее процессом постоянного пересмотра и переосмысления. И, отметил бы, что в научном дискурсе существуют иерархии знаний, властные структуры и доминирующие интерпретации, которые формируют искаженное представление о самом понятии науки!

        Ну, да, ладно, не об этом речь! Вся эта динамика и эволюция понятий пропитанные как грязное бельё властью... Я не буду тут полоскать идеи постструктуралистов, их бесплотные ковыряния в мыслях и словах. Я вот о чем...

        Ради чего вся эта научная канитель, эти призывы к рациональности? Вот вопрос!

        Вспоминается отрывок из сюжет фильма "Ярость". Там командир танка со своими подопечными и поддержкой пехоты захватывают некий город и очищают его, понятно каким образом, от врагов...

        И дальше интересно... Всё затихло! Враг сгинул! Привал, короче! И командир, пока солдатики еб*т какую-то местную бабенку в своём танке на потоке, поднимается в уцелевшее здание вместе с новобранцем из его команды и встречает двух спрятавшихся в квартире барышней, новобранец даже подмечает их красоту и начинает испытывать симпатию к одной из них!

        Герой спрашивает, если у них куриные яйца, и на утвердительный ответ просит приготовить яичницу! Вообщем они все садятся за обеденный стол и надеяться мирно потрапезничать, как дома, но приходят их два товарища и начинают хамить и в целом надсмехаться над этим "цирком"!

        Но, командир, тыкая в каждого из них пальцем, говорит что им не испортить этот момент как бы им этого не хотелось и чтобы они об этом не думали. Потомучто он хочет мирно и культурно потрапезничать! Он заканчивает трапезу в молчании товарищей и они уходят!

        Да, потом прилетает бомба, дом в щепки, новобранец бежит к обломками надеясь спасти дамочку, но его хватает за шею его же товарищ со словами "куда ты побежал, ты ещё не понял - это война"!

        И это очень глубокая сцена! Ведь в ней осмысленно, только командир понимает для чего вся эта вакханалия боевых действий! Ради мира и защиты культуры! Ради того что бы вместе потрапезничать за спокойной беседой, наслаждаясь моментом!

        Но, его товарищи уже не таковы! Война у них внутри! Она уже просто как они выражаются в фильме "лучшая работы в мире"!

        Так о чем мы собственно? Ах, да... Наука!

        Вот наука - это собака, которая себя чем-то возомнила! А, собака, как знают кинологи, должна знать свое место, иначе у тебя возникнуть могут большие проблемы! И, если собака забывается, то ей надо указать место и как можно быстрее! Чтобы она не забывала, кто здесь настоящий хозяин и вокруг чего должен вращаеться её мир! И тогда и хозяину и собаке хорошо вместе, радостно!

        И вот сегодня наука дошла то такой борзоты... ой, выражусь политкорректно... дошла до такого этапа эпистемиологической эволюции... что для проверки её практичности приходится тратить миллиарды баксов!

        Все эти сливы бабок, которые оплачивает, между прочим, если кто забыл, население, мы! Под подпевания меркантильных учёных, желающих кататься на мерседесах, живя в хорошем доме со своими детишечками, и не против и дальше, хоть 100 лет на пролёт, нажимать кнопочку Enter в своём Церне, чтобы процессорики пару дней повысчитывали уравнения стандартной модели, пока они покурят на свежем воздухе и потравят анекдотики! А, в это время где-то в Мухосранске умирает бабушка в больничке из-за отсутствия копеечного "Нитроглицерина"!

        Я, конечно же, утрирую! Но, меня вот не спросили, одобряю ли я слив миллиардов, на то чтобы узнать, что произойдёт при столкновении двух каких-то еб*чих протонов!

        Но, если выбирать между лицезрением глаз родителей больного ребёнка, выслушивающим цены на лечение их дитя, и выяснением как расположить хров детектор в коллайдере, чтобы точнее замерить спектр бозона Хигса, то ответ будет однозначный: "Идите нхуй куда подальше со своим Хигсом!!".

        Ну, у нас всё это так, будто где-то в другом мире происходит и как будто Божия благодать снисходит и оплачивает все счета увлеченнных и стремящиеся к просвещению научных сотрудников, в опыта которых происходит просто некая мана небесная, развенчания чуда на ладонях!

        А, что внутри-то нас? Пока мы изучаем атомы и строим по факту ядерные вооружения! Если наука как охранная собака! Если, бл*ть, все такие образованные и разумные до невозможности!

        Для этого достаточно включить европейский канал и беседу прозападного журналист с каким-нибудь находящегося в опале чиновником другой страны! И кроме обмена угроз мы ничего хорошего не услышим... Мы убъем всех будет орать один, у нас столько-то бомб скажет другой, будете делать как мы сказали заметит третий, и т.п.

        Но, конечно, это просто людское недоразумения! Надо просто собраться и рассказать близким братьям, что все дело в квантовом детерминизме, надо мыслить рационально, и главное отличать и не путать рациональное мышление и рациональное поведение, вот!

        Но, правда в том, что никакие призывы, никакие слова здесь не помогут... Вопрос, как избавить человека от чувства, когда он приходит после работы и смотрит в зеркало, и осознает, живя в сегодняшнем мире, что всем на него наплевать! Что везде почти он как разменная монета! И весь пздешь о равенстве, свободе и братстве заканчивается работой за гроши на какое-то хамло, параллельно выслушивая от знакомых, что просто долбеб и давно бы взял кредит в воровских и набывающих банках и стал бы главным бизнесменом страны, а не протирающим подштанники лхом!

        Отчуждение! Отчуждение от человека! Вот подарок просвещения, вокруг которого как с писанной торбой носятся апологеты науки аплодируя чуть ли не на коленях! Благо экзистенциализм немного остудил пыл этих думающих что они рациональные "машины", учёных!

        И, спасибо, Фуко, Делёзу и Дерриды и др, а также Лакану, Жижеку и др что не дают окончательно превратиться этим ученым методичкам в пресловутую тоталитарную идеологию! Спасибо, дорогие!

        Поглядим, что будет дальше, но науке надо хвост поприжать!

        Да, и напоследок специально для вас, Диметор!

        Вклад в историю вносят различные люди, независимо от их психического состояния. Психическое здоровье или особенности не определяют культурные, научные или исторические достижения.

        Важно помнить, что каждый человек уникален, забудьте о своей статистике и корреляция сейчас, и его вклад в общество и историю определяется множеством факторов, включая индивидуальные способности, обстоятельства жизни и социокультурный контекст. И даже если этого вклада нет с его стороны, он все равно бесценен!

        Да, уж... Бесценен? ! Сам пишу и сам же смеюсь! Так бы и сказал: "Боже спаси нас, не ведуем, что творим! Но, слава Богу, я атеист!


  1. Proscrito
    13.04.2024 11:24

    На мой взгляд немного лишнее противопоставление случайности и фатализма. Фатализм не зиждется на возможности предвидеть все квантовые флуктуации от планковской эпохи до тепловой смерти. Насколько я его понимаю, фатализм утверждает отсутствие выбора (свободы выбора) и квантовый хаос не вступает с ним в противоречие. Пусть наша судьба зависит не от изначальных условий, но от случайности, кому от этого легче? :) Противники фатализма в основном отстаивают идею свободы воли, а ей нет места ни в супердетерминированном, ни в абсолютно случайном мире.

    И несколько не хватает объяснения того, почему наблюдения 13.8 млрд лет эволюции Вселенной настолько хорошо согласуются с принципом детерминизма. По крайне мере для "макро" масштаба. Как так выходит, что мы не в силах описать поведение частицы и на квант времени, но поведение планет - запросто на эоны. И нигде на масштабах Вселенной не было обнаружено ни одного объекта, для объяснения состояния которого сегодня, потребовался бы эффект бабочки где-то в прошлом.

    Кстати, эффект бабочки - это термин из теории динамического хаоса и здесь нужна отдельная дискуссия о том, что такое стохастические системы и как они согласуются (или не согласуются) с детерминизмом. Здесь, кстати, можно привести в пример "злободневную" "проблему трех тел". Предсказать поведение системы из трех звезд можно лишь с некоторой точностью и до некоторого момента, аналитического решения для общего случая не существует, система стохастическая. Без всякого "истинного" квантового хаоса. Но нарушает ли такая система принцип детерминизма? Нет. Существует множество частных решений, а наша возможность предсказывать упирается только в доступную точность измерений входных параметров. Система - идеальный кандидат для демонстрации эффекта бабочки, при этом никаким индетерминизмом в ней не пахнет.

    В целом статья интересная. Но чего-то не хватает :)


    1. dionisdimetor Автор
      13.04.2024 11:24

      Противники фатализма в основном отстаивают идею свободы воли, а ей нет места ни в супердетерминированном, ни в абсолютно случайном мире.

      Я тоже не верю в свободу воли, но и фатализм не принимаю. Не может быть судьбы в мире, где есть квантовая случайность. Фатализм предполагает, что от нас вообще ничего не зависит, что бы мы не делали - исход предопределён. Но если ничего не предопределено, наши действия на что-то, да влияют. Неважно, супердетерминированы они или случайны - если их нельзя точно предсказать, мы вполне можем считать свои решения свободными. Тем более, когда мы способны к саморефлексии и самопрогнозированию.

      Как так выходит, что мы не в силах описать поведение частицы и на квант времени, но поведение планет - запросто на эоны.

      Да, интересная тема: откуда берётся устойчивость макрообъектов на фоне квантовых флуктуаций, как возникает порядок из хаоса и насколько детерминирован эффект бабочки. В одну статью это явно не уложишь, но я как раз собираюсь написать и о теории хаоса, и о свободе воли.


      1. Proscrito
        13.04.2024 11:24

        Из того, что мы не можем предсказать будущее (неважно, в силу которого из хаосов) не следует, что мы на что-то влияем. В "лучшем случае" судьба зависит от броска монетки, но все равно не от "нас". Пока что никаких "нас" найти не получается ни в детерминированной, ни в хаотической Вселенной. В любой из них мы только статисты.

        Кстати, сознание и способность к рефлексии - тоже сугубо наблюдательные функции. Как показывают эксперименты, мы сперва делаем выбор, а только потом его осознаем и объясняем.

        Статью буду ждать )


        1. dionisdimetor Автор
          13.04.2024 11:24

          Даже если свободы воли нет и единого-неделимого "я" тоже, само по себе осознание того факта, что наше поведение недетерминировано, уже влияет на это поведение, причём вполне детерминировано:) Да и с точки зрения эволюции - как могло сформироваться самосознание, если бы оно имело только наблюдательную функцию?


          1. Proscrito
            13.04.2024 11:24

            Один из тех внешне простых вопросов, ответ на который можно дать разве что курсом лекций Сапольски )

            Что бы ни влияло на наше поведение, мышление - это достаточно простой с точки зрения химии и физики процесс, без всякого квантового хаоса, и уж тем более мистической духовной составляющей. Если не брать во внимание конечно маргинальную теорию квантового сознания. Этот процесс имеет вполне себе детерминистическую природу. Электрический сигнал бежит по синапсам, нейромедиаторы передают его в межсинаптическом пространстве, все довольно прозаично и никакой свободы воли там не просматривается.

            Аргумент про эволюцию не очень понятен. Да, мы не единственные животные, обладающие сознанием. Более того, оно присуще гораздо большему количеству видов, чем можно подумать. Однако, далеко не всем. Даже среди эукариот. Даже среди животных. Полным-полно видов жизни, которые прекрасно эволюционируют без сознания вообще. Эволюция это как раз аргумент против свободы воли (впрочем, аргументов за я и не знаю), она указывает на то, что любые жизненные формы подвержены естественному отбору, и сознание это всего-навсего еще один признак усложнения. Нужна для этого свобода воли? Нет.

            Я уже упоминал, что мы принимаем решения до их осознания. Это достаточно неплохо установленный факт, начиная с опытов Либета в начале 80х. Поэтому где бы вы ни искали "двигатель" свободы воли, сознание сюда не подходит ) "Воля" проявляется раньше, чем вступает в дело сознание.

            Зачем оно нужно? Ну как раз для рефлексии. Нейросеть наша обучается. И обучается она лучше, если может прогонять для обучения один и тот же датасет, варьируя варианты и моделируя ответы. Для этого нам память, опыт, возможность рефлексировать, размышлять над абстрактными проблемам и многое другое, что умеет наш мозг. Но по факту всем этим мы просто обучаем свою нейросеть, чтобы когда пришло время решать, она выдала наилучший результат. Который мы затем также можем подвергнуть анализу и дообучить нашу нейронку с учетом вновь полученного опыта. Однако никакой свободы воли там нет, процесс принятия решения абсолютно "механический".

            Осознание чего-либо - это дообучение нейросети. Осознавая что-то наш мозг физически изменяется. Он иным образом реагирует, выдает другие результаты. Осознание меняет его, как и любая другая новая информация. Поэтому, в силу принципа причинности, осознание действительно влияет на будущее. Только свобода воли потерялась в этом процессе ) Зато причинность заиграла другими красками.


            1. dionisdimetor Автор
              13.04.2024 11:24

              Свобода воли как возможность независимо от обстоятельств делать выбор является свойством целостного субъекта, а наш мозг таковым не является, отсюда и все разногласия. Допустим, вы верили в свободу воли, а потом посмотрели лекции Сапольски и перестали верить. Это было детерминировано? Нет. А где гарантия, что я или кто-то другой не смогут вас переубедить? Наши убеждения не определены ни генами, ни личным опытом. Возможно, они являются результатом естественного отбора среди идей, если верить теории нейродарвинизма Дж. Эдельмана. Аргументы - тоже идеи (мемы), под влиянием которых мы можем изменить, а можем не изменить своё мнение. Более того, части одного мозга могут иметь разные мнения и спорить между собой. И когда мнения распределяются примерно поровну, в игру вступает квантовая случайность. А ещё наш мозг использует не чёрно-белые категории, а субъективные вероятности и нечёткую логику. Короче, всё сложнее, чем кажется на первый взгляд.


              1. Proscrito
                13.04.2024 11:24
                +1

                Стоп. Если я верил в свободу воли, а после лекции Сапольски перестал, то почему это не было детерминировано? Я не понял. Было конечно. Если комета летела с вектором скорости V и внезапно столкнувшись с астероидом изменила этот вектор, это что, не было детерменированно? Было. Даже если мы этого не могли рассчитать, потому что на комету и астероид в этот момент действовало несколько мощных полей тяготения и нам не дано было точно определить вероятность столкновения.

                Вообще по барабану, какую там логику использует наш мозг. Размышления и логика - это всего лишь модели абстракции. Для чего их использует мозг - я написал выше. И технически дообучение мозга ничем не отличается от встречи с астероидом. Никакой там свободы воли не нужно, это излишняя сущность. Самый тот случай, где стоит использовать бритву.

                Все однозначно сложнее, чем кажется, но неужели вы думаете, что первый задаетесь подобными вопросами? :) Абсолютно все логические финты, рефлексия, самоанализ, споры с самим собой и прочие приемы - это способы переобучить сеть. Есть ли свобода воли у теплового реле? Оно принимает решение разомкнуть, или сомкнуть цепь, в зависимости от нагрева пластины. И со временем из-за неупругих деформаций, усталости металла, оксидной пленки и прочих факторов, этот диапазон температур может меняться. Нужна тут свобода воли? Нет.

                Вас путает кажущаяся сложность поведения. У реле только 2 функции, у нашего мозга - миллионы. Но это количественное различие, а не качественное.

                Да, про квантовую случайность в решениях. Во-первых это антинаучное предположение. Оно нефальсифицируемо, нет смысла его обсуждать. Во-вторых оно не показывает свободу воли. Я уже об этом выше говорил. Даже если квантовый хаос как-то вмешивается в механизм принятия решений, то это не воля, ни в одном из известных определений воли. Это чистая случайность. Тогда и любой другой естественный процесс, в котором замешана квантовая случайность, можно назвать проявлением воли.


  1. SergioShpadi
    13.04.2024 11:24

    Зная историю швейцарской криптографии, я бы побоялся у них покупать генератор «истинно случайных» чисел. Вдруг там, скажем так, встроенный супердетерминизм.


    1. dionisdimetor Автор
      13.04.2024 11:24

      Ха, так вот почему Жизан пропагандирует квантовую случайность - он же швейцарец! И в ЦЕРНе все в курсе заговора, просто не признаются. Тогда остаётся надеяться на Galaxy Quantum - может хоть китайский смартфон генерирует истинно случайные числа:)